Anda di halaman 1dari 64

Universidad La Salle.

Facultad Mexicana de Medicina Curso de Extensin Universitaria para la Preparacin del Examen Nacional para Aspirantes a Residencias Mdicas. Examen Final. 3 Septiembre de 2008. Modalidad a Distancia
1. De acuerdo a la Clasificacin de fracturas expuestas de Gustillo, una fractura con herida cutnea mayor de 1cm y una lesin moderada de partes blandas, corresponde a una tipo: a) I b) II c) IIIA d) IIIB Las fracturas abiertas se clasifican utilizando el esquema de Gustillo y Anderson, que distingue tres grandes tipos: El tipo I es resultado de traumatismos de menor energa, generalmente indirectos, y la herida se produce porque uno de los extremos seos fracturados corta los tejidos desde el interior del miembro hacia el exterior. Las heridas suelen medir menos de 1 cm, su riesgo de complicaciones es muy bajo y pueden tratarse como fracturas cerradas.

Tipo II- implican una herida mayor de 1cm de largo, as como una lesin significativamente

mayor, provocada por el incremento de la absorcin de energa cuando se produce la fractura, lo usual es que tengan cierto grado de conminucin y un componente de aplastamiento mnimo a moderado. Las fracturas abiertas resultantes de traumatismos de alta energa suelen tener gran destruccin de partes blandas y se denominan tipo III; la longitud de la herida suele ser mayor de 10 cm y sobre todo llama la atencin la gran destruccin de partes blandas. A su vez, incluye tres subtipos: III A. Es posible suturar las partes blandas lesionadas hasta cubrir por completo el hueso sin dejar ninguna ventana en la que la superficie sea comunique directamente con el exterior. III B. Para no dejar hueso expuesto, es necesario aplicar procedimientos de cobertura como los colgajos e injertos. III C. Existe una lesin vascular asociada a la fractura abierta, lo cual empeora enormemente la gravedad de la lesin y aumenta las probabilidades de requerir amputacin. Skinner, H. Diagnstico y tratamiento en Ortopedia. Ed. Manual Moderno. Mxico, 2004. pp. 91

2. La maniobra que nos indica en un recin nacido, que la articulacin de la cadera esta luxada, se conoce como: a) Signo de Galeazzi b) Signo de Barlow c) Signo de Ortolani d) Signo de Pistn

Prueba de Ortolani : con esta prueba se detecta una cadera ya luxada, se coge con la mano el miembro flexionado, la cadera se coloca en abduccin mientra se levanta el fmur con cuidado y se sita los dedos a nivel del trocnter mayor. Si la prueba es positiva se siente la reduccin de la cadera dentro del acetbulo. Prueba de Barlow: es una prueba inductora para identificar una cadera inestable pero an localizada en su sitio; no es una prueba adecuada para diagnosticar luxacin de cadera. La cadera en aduccin ligera y con la palma de la mano se empuja suave y cuidadosamente hacia atrs, la presencia de un movimiento de pistn o al percepcin de una cabeza femoral subluxada sobre el borde posterior del acetbulo. Prueba de Galeazzi: con el nio acostado se le flexionan las caderas y rodillas de modo que los talones se apoyen sobre la mesa y reconocer el acortamiento relativo del muslo.
Skinner, H. Diagnstico y tratamiento en Ortopedia. Ed. Manual Moderno. Mxico, 2004. pp. 625

3. El signo clnico de la charretera nos orienta a pensar en una luxacin glenohumeral: a) Anterior b) Posterior c) Superior d) Axilar Luxacin glenohumeral Anterior: Es la forma ms frecuente de luxacin de hombro y una de las ms frecuentes del organismo. El paciente acude con el brazo en discreta separacin y rotacin externa fijas: cualquier intento de movilizacin del hombro resulta doloroso. A la inspeccin se aprecia la llamada deformidad en charretera, por el afilamiento acromial que presenta el hombro al desaparecer el perfil redondeado de la cabeza humeral Manual CTO 7 edicin, Traumatologa y Ortopedia, pg 20

4. El signo radiogrfico de Thurston Holland es caracterstico de las lesiones fisiarias tipo (Salter y Harris): a) I b) II

c) III d) IV Siguiendo la clasificacin de Salter y Harris, podemos distinguir cinco tipos de epifisilisis. 1) Tipo I. El trazo lesional cursa en su totalidad por la fisis (zona hipertrfica profunda). 2) Tipo II. El trazo cursa por la fisis pero asciende hacia la metfisis desprendiendo un fragmento metafisario triangular. 3) Tipo III. El trazo provoca la discontinuidad de la epfisis y prosigue a travs de la fisis. 4) Tipo IV. El trazo provoca discontinuidad de la epfisis y asciende hacia la metfisis desprendiendo un fragmento metafisario triangular como en las tipo II. Es tpica del cndilo humeral lateral infantil. 5) Tipo V. Compresin axial con destruccin de las zonas de reserva y proliferativa. Manual CTO 5 edicin, Traumatologa y Ortopedia, Pg. 38

5. La maniobra para efectuar la reduccin de la subluxacin de cabeza de radio en nios debe ser en: a) Extensin y pronacin de codo b) Flexin y pronacin de codo c) Extensin y supinacin de codo d) Flexin y supinacin de codo Subluxacin de la cabeza del radio fuera del ligamento anular, que ocurre en nios de entre uno y tres aos como resultado de una traccin axial sobre el miembro superior Cursa con dolor, codo en semiextensin, antebrazo en pronacin y ausencia de movilidad activa de la extremidad superior afecta. El tratamiento consiste en supinar forzadamente el antebrazo en extensin y a continuacin flexionarlo. Manual CTO 7 edicin, Traumatologa y Ortopedia, Pg. 37

6.- Cul es el tratamiento de eleccin para la escarlatina?: a) b) c) d) Penicilina benzatnica. Cefuroxime. Amoxicilina. Eritromicina.

Streptococcus pyogenes (hemoltico del grupo A) es el agente etiolgico de la escarlatina, no ofrece resistencia a la penicilina benzatnica y con dosis nica ofrece un buen ndice de curacin, erradicacin y reduccin de complicaciones. Gonzlez-Saldaa N, Infectologa Clnica Peditrica, 7 edicin, pginas 403-407

7.- Las complicaciones tardas o no supuradas de la escarlatina son? a) b) c) d) Adenitis cervical y absceso cervical. Otitis media aguda y sinusitis. Fiebre reumtica y glomerulonefritis. Celulitis y fascitis necrosante.

Algunas cepas de Streptococcus pyogenes (hemoltico del grupo A) son cardiognicas o nefrognicas, que mediante mecanismos inmunitarios posterior a la infeccin desencadenan fiebre reumtica y glomerulonefritis. Gonzlez-Saldaa N, Infectologa Clnica Peditrica, 7 edicin, pginas 403-407. 8.- Cul es la definicin de otitis media recurrente? a) Cuadros repetidos de otitis media que alternan sin periodos de remisin, 3 o ms cuadros en 6 meses, o 4 o ms cuadros en 1 ao. b) Presencia de signos y sntomas de ms de 3 meses de duracin, con perforacin del tmpano. c) Cuadros repetidos de otitis media que alternan con periodos de remisin evidente, 3 o ms cuadros en 12 meses, o 4 o ms cuadros en 2 aos. d) Cuadros repetidos de otitis media que alternan con periodos de remisin evidente, 3 o ms cuadros en 6 meses, o 4 o ms cuadros en 1 ao.

La definicin establecida para recurrencia requiere: 1) remisin evidente entre los cuados y 2) una frecuencia mnima de 3 o ms cuadros en 6 meses, o 4 o ms cuadros en un ao. Gonzlez-Saldaa N, Infectologa Clnica Peditrica, 7 edicin, pginas 63-98.

9.- El agente causal ms frecuente de la otitis media aguda es? a) b) c) d) Haemophilus influenza tipo B. Streptococcus pneumoniae. Staphylococcus aureus. Moraxella catarrhalis.

El Streptococcus pneumoniae representa del 30 al 35 % de los casos de otitis media aguda desde las revisiones de Feigin (1981), Ruuskanen y Heikkinen (1994) y Blumer (1997).

Gonzlez-Saldaa N, Infectologa Clnica Peditrica, 7 edicin, pginas 63-98.

10.- Para el diagnstico radiolgico de sinusitis aguda, la proyeccin que mejor valora los senos maxilares y las estructuras intranasales es? a) b) c) d) Lateral. Anteroposterior. Submentoniana. Waters.

La proyeccin Waters es la proyeccin que mejor permite valorar la neumatizacin, opacificacin o engrosamiento de mucosa de los senos maxilares as como las estructuras intranasales. Gonzlez-Saldaa N, Infectologa Clnica Peditrica, 7 edicin, pginas 63-98.

11- En la tosferina se presentan accesos de tos paroxstica, acompaada de cianosis, apnea y periodos de apnea durante la fase: a) b) c) d) Periodo catarral. Periodo paroxstico. Periodo de convalecencia. Periodo de curacin.

Durante el periodo paroxstico con una duracin de 2 a 4 semanas, son caractersticos los accesos de tos paroxstica, acompaada de cianosis, apnea y periodos de apnea durante la fase paroxstica. Gonzlez-Saldaa N, Infectologa Clnica Peditrica, 7 edicin, pginas 145-156.

12.- La meningitis bacteriana presenta alteraciones en el lquido cefalorraqudeo, tales como: a) Agua de roca, protenas de 15 a 45 mg/dl, clulas de 10, glucosa de 1/2 a 1/3 de la glucemia central. b) Turbio, protenas > 45 mg/dl, clulas de > 10, predominio polimorfonucleares, glucosa < de 1/2 a 1/3 de la glucemia central. c) Agua de roca, protenas > 45 mg/dl, clulas de > 10, predominio mononucleares, glucosa < de 1/2 a 1/3 de la glucemia central. d) Xantocrmico, protenas > 100 mg/dl (muy altas), clulas de 100 a 500 (rara vez ms), predominio mononucleares, glucosa < de 1/2 a 1/3 de la glucemia central. El diagnstico de meningitis bacteriana requiere del estudio inicial de lquido cefalorraqudeo, en el cual se muestra turbidez del lquido, proteinorraquia moderada,

celularidad que va desde 100 hasta incontables, pero con predominio de polimorfonucleares, aunado al la hipoglucorraquia dado el metabolismo bacteriano y cierra de la barrera hemateenceflica (meningitis). Gonzlez-Saldaa N, Infectologa Clnica Peditrica, 7 edicin, pginas 245-260.

13. Sofa de 24 aos G1 P1, acude al servicio mdico por referir, una secrecin transvaginal bastante lquida, de baja viscosidad, maloliente de color amarillo y gris, espumosa. Esta entidad es propia de infeccin por: a) Cndida albicans b) Gardnerella c) Tricomonas d) Gonococos

Tricomoniasis

Es causada por el Trichomonas vaginalis, parsito protozoario unicelular de cola tipo ltigo que utiliza para impulsarse a travs del moco de la vagina y la uretra. ETS que se presenta a nivel mundial. Sntomas de la tricomoniasis Los sntomas de la enfermedad son bastante diferentes en hombres y mujeres: En los hombres, la infeccin es asintomtica(no tiene sntomas) y desaparece espontneamente en algunas semanas. Pero en caso de presentar sntomas, son los siguientes:

Picazn uretral leve o secrecin. Ardor ligero despus de orinar o eyacular. Pueden presentar prostatitis o epidimitis por la infeccin.

En las mujeres:

Secrecin vaginal espumosa blanca-verde o amarillenta, de olor ftido. Cantidad considerable de secrecin que ocasiona picazn en los labios y parte interna de los muslos y labios, parte que tambin pueden inflamarse.

Incomodidad durante el coito.

Es una infeccin frecuente, ms de la mitad de mujeres con gonorrea tambin presentan tricomonas, ya que con frecuencia las personas con una enfermedad de transmisin sexual diagnosticada, tienen una o mas infecciones transmitidas sexualmente.

. Krieger JN and Alderete JF. Trichomonas vaginalis and trichomoniasis. In: K. Holmes, P. Markh, P. Sparling et al (eds). Sexually Transmitted Diseases, 3rd Edition. New York: McGraw-Hill, 1999, 587-604.

14. El tratamiento de eleccin para esta entidad es: a) Metronidazol 500 mg. VO c/12 por 7 a 10 das b) Ampicilina 1g VO c/ 6 hrs. c) Clotrimazol + Metronidazol 500 mg VO c/12 hrs. por 7 a das d) Clindamicina 300 mg VO cada 12 hrs. x 5 das.

La tricomoniasis suele curarse con una dosis oral nica del medicamento metronidazol, el cual se vende con receta. Los sntomas de la tricomoniasis en hombres infectados puede que desaparezcan en unas cuantas semanas sin tratamiento alguno. Sin embargo, un hombre infectado, aun cuando nunca haya tenido sntomas o cuyos sntomas hayan desaparecido, todava puede continuar infectando o re-infectando a su pareja sexual hasta que no se trate la infeccin. Por lo tanto, ambos en la pareja deben tratarse la infeccin al mismo tiempo para eliminar el parsito. Las personas que se estn tratando la tricomoniasis deben evitar tener relaciones sexuales hasta que tanto ellas como sus parejas hayan terminado el tratamiento y no tengan sntomas. Las mujeres embarazadas pueden tomar metronidazol. Krieger JN and Alderete JF. Trichomonas vaginalis and trichomoniasis. In: K. Holmes, P. Markh, P. Sparling et al (eds). Sexually Transmitted Diseases, 3rd Edition. New York: McGraw-Hill, 1999, 587-604.

15. El frmaco de eleccin para el tratamiento de Gardnerella es: a) Metronidazol b) Miconazol

c) Clotrimazol d) Tetraciclinas De manera ideal el tratamiento de Gardnerella vaginalis debe inhibir el crecimiento de las bacterias anaerobias pero no de los lactobacilos vaginales. Por lo que el tratamiento de eleccin es el Metronidazol a dosis de 500 mg VO dos veces al da durante siete das. Y como rgimen alternativo consiste en una sola dosis oral de 2 gr de Metronidazol. DeCherney A. Enfermedades benignas de la vulva y la vagina. Diagnstico y tratamiento ginecoobsttricos. 7 ed. Mxico. Manual Moderno. 2000.

16. En un frotis vaginal la presencia al microscopio de clulas clave son propias de la infeccin por: a) Cndida albicans b) Gardnerella c) Tricomonas d) Gonococos

La infeccin por gardnerella (bacteria) tiende a producir una secrecin blanca, gris o de color amarillo turbio, con un olor ftido o a "pescado" que aumenta cuando la secrecin se vuelve alcalina, como sucede despus del coito o de lavarse con jabn. Puede haber prurito o irritacin vulvar, pero por lo general no son muy pronunciados. Al microscopio se aprecia las famosas clulas clave. Realizando el diagnstico diferencial.

Pernoll M. Enfermedades de trasmisin sexual. En: Manual de Obstetricia y Ginecologa. Benson/Pernoll Editores.Editorial Interamericana S A. Mxico DF, 1994

17. Cul de las siguientes substancias es la adecuada para el tratamiento del Hirsutismo asociado al sndrome ovrico poliqustico? a) Clomifeno b) Estrgenos c) Acetato de ciproterona d) Corticoide

El acetato de ciproterona parece ser ms efectivo que otros frmacos para el hirsutismo en mujeres causado por la produccin ovrica excesiva de andrgenos

Una de las causas de hirsutismo (crecimiento piloso excesivo) en mujeres es la hiperproduccin de andrgenos a partir del ovario. Varios frmacos pueden utilizarse para contrarrestar los efectos del andrgeno. El acetato de ciproterona es un frmaco antiandrognico. Los efectos adversos informados con su uso fueron aumento de peso, depresin, fatiga, sntomas mamarios y disfuncin sexual. La revisin de los ensayos encontr que el acetato de ciproterona parece ejercer un efecto en el hirsutismo similar a otros frmacos utilizados para el tratamiento del hirsutismo por exceso de andrgenos. No existen pruebas suficientes para comparar los efectos adversos de las opciones de tratamiento.

Van der Spuy ZM, le Roux PA. Acetato de ciproterona para el hirsutismo (Revisin Cochrane traducida). En: La Biblioteca Cochrane Plus, nmero 4, 2007. Oxford, Update Software Ltd. Disponible en: http://www.update-software.com. (Traducida de The Cochrane Library, 2007 Issue 4. Chichester, UK: John Wiley & Sons, Ltd.).

18. El sntoma caracterstico de la endometriosis es? a) Dispareunia b) Metrorragia c) Leucorrea d) Dismenorrea

Los sntomas ms comunes son: Dolor agudo en la pelvis durante el coito Dolor que aumenta progresivamente con el tiempo y comienza cada vez ms temprano antes de la menstruacin Dolor severo que se agudiza en los ltimos das de la menstruacin Dificultad o imposibilidad para quedar embarazada

El-Eoley, et al. Danazol but not ginadotropin releasing hormone agonists suppresses autoantibodies in endomeriosis. Fertil Steril 1990; 54:725

19. Para realizar el diagnstico preciso de endometriosis se requiere? a) Visualizacin directa de las lesiones patognomnicas b) Corroboracin histolgica de glndulas y estroma en tejido obtenido por biopsia de las lesiones sospechosas observndolas directamente por laparoscopia o laparotoma c) Deteccin ecogrfica de un endometrioma d) Presencia de masa dolorosa en paciente frtil

La endometriosis es una condicin que se caracteriza por la presencia de sntomas variados. Lesiones mnimas pueden estar asociadas con dolor severo, y por el contrario lesiones extensas pueden permanecer asintomticas. Las formas infiltrantes de la endometriosis han sido descritas desde 1913. En esta forma particular de la enfermedad la endometriosis penetra por debajo de la superficie del peritoneo y puede no ser visible. Las lesiones de endometriosis que invaden ms all de cinco mm se definen como profundamente invasivas. 10 Debido a su localizacin subperitoneal, el diagnstico es con frecuencia difcil, incluso por va laparoscpica. 11 Estas formas infiltrantes estn estrechamente relacionadas con dolor plvico y dispareunia profunda. Asimismo, las lesiones estn generalmente localizadas en el fondo del saco de Douglas, ligamentos uterosacros y pliegue tero vesical. Junto con los endometriomas ovricos, estas lesiones son consideradas la manifestacin ms severa de la enfermedad.

Mangal R, Taskin O, Nezhat C, Franklin Robert. J Reprod Med 1995; 41: 64-6. Goh JT, Flynn V. Inguinal endometriosis. Auzt NZ J Obstet Gynecol 1994; 34: 121.

20. Cul de las siguientes estructuras resultan afectadas con mayor frecuencia por la endometriosis? a) Ovarios, cuello uterino y vagina b) Ovarios, fondo de saco posterior y trompas c) Ovarios, pliegue tero vesical anterior y fondo de saco posterior d) Colon sigmoide, ovarios y tero

La endometriosis se encuentra con ms frecuencia en los ovarios, pero tambin se pueden encontrar en otros lugares (indicados arriba en color prpura -morado-), incluyendo los siguientes: Las trompas de Falopio. Los ligamentos que sostienen el tero. El rea interna entre la vagina y el recto. La superficie externa del tero. En el revestimiento de la cavidad plvica.

Ocasionalmente, los implantes se encuentran en otros lugares, como lo son: Los intestinos. El recto. La vejiga. La vagina. El cervix. La vulva. Las cicatrices de cirugas abdominales.

Han sido descritos 37 casos de endometriosis inguinal desde 1896. 4 Los cirujanos y gineclogos tratantes de esta condicin usualmente describen una masa inguinal que puede ser dolorosa y que flucta durante la menstruacin. Algunos informes han indicado que las lesiones intraplvicas no fueron encontradas en estas pacientes durante la realizacin de la ciruga laparoscpica. 5-7 En otros casos, la ciruga no fue realizada debido a la falta de sntomas ginecolgicos. 8 Sin embargo, Candiani y cols. informaron de seis pacientes con endometriosis, tanto inguinal como intraplvica, profundamente infiltrante. Las lesiones inguinales descritas estuvieron localizadas en la porcin extraperitoneal del ligamento redondo, sin ninguna comunicacin con la pelvis. Los autores recomendaron la escisin completa de la endometriosis inguinal para evitar la diseminacin, as como tambin, la realizacin de laparoscopia diagnstica concomitante, debido a la alta asociacin de sta, con la presencia de endometriosis intraplvica.

Mangal R, Taskin O, Nezhat C, Franklin Robert. J Reprod Med 1995; 41: 64-6. Goh JT, Flynn V. Inguinal endometriosis. Auzt NZ J Obstet Gynecol 1994; 34: 121.

21. La ovulacin en la especie humana ocurre generalmente? a) b) c) d) Dentro de las 24 horas previas al pico de LH Coincidiendo con el pico de LH Dentro de las 24 horas posteriores al pico de LH Coincidiendo con el pico de estradiol

Otro mtodo para detectar la ovulacin es a travs de la medicin de una hormona llamada LH; dicha hormona LH es la responsable que se produzca la ovulacin y la misma ocurre 24 hs. despus de un brusco ascenso de esta hormona. La hormona LH es detectable tanto en sangre como en orina y a travs de un mtodo de medicin de LH en orina se puede detectar su presencia. El da que dicho anlisis da positivo significa que al da siguiente la mujer va a ovular. Hay otros mtodos para la deteccin de la ovulacin pero que ya escapan al manejo casero de los mismos; estos son fundamentalmente la medicin de hormonas por ejemplo: la progesterona y la observacin del crecimiento del folculo a travs de la ecografa. Rodrigues I, Grou F, Joly J. Effectiveness of emergency contraceptive pills between 72 and 120 hours after unprotected sexual intercourse. Am J Obstet Gynecol 2001;184:531-537. 22. La toma de anticonceptivos orales combinados por tiempo prolongado disminuye el riesgo de padecer?

a) b) c) d)

Adenoma hepatocelular Melanoma Adenocarcinoma endometrial Ca. epidermoide del cerviz

Muchos autores 1,3,4 han precisado factores de riesgo y predisponentes que contribuyen a la aparicin de esta enfermedad, las estadsticas sealan la baja paridad o infertilidad, la presencia de ciclos anovuladores, los trastornos menstruales, la menopausia tarda, las hemorragias disfuncionales en la menopausia, los tumores funcionantes del ovario; el uso de estrgeno exgeno; enfermedades como obesidad; diabetes e hipertensin arterial. Se acepta que en la gnesis del carcinoma de endometrio desempea un papel importante la existencia de estmulo estrognico mantenido no equilibrado con una secrecin de progesterona. Averette HE, Nguyen H. Cncer ginecolgico en Oncologa Clnica. Manual de la American Cncer Society. 2. ed. Lawrence:OPS;1996: b23-b26.(Publicacin Cientfica, No.559).

23. Cul es el promedio de sobrevida en la trisoma 13? a) 4 meses b) 12 meses c) 6 meses d) 1 mes La sobrevida en esta enfermedad es baja: 55% de los pacientes fallecen en el primer mes de vida, 25% muere entre el mes y los 2.5 meses, despus de esta edad y antes De los seis meses fallecen 15% y slo 5% mueren despus de esta edad. El promedio de vida para el sexo masculino es de cuatro meses y el del femenino es de 20 meses. En cuanto a la principal causa de muerte, son las malformaciones cardiacas, tal como aconteci en Este caso, y sus complicaciones, entre las que destaca la apnea recurrente.

Fuentes F. Sndrome de Patau (Trisoma 13); An Espaoles Pediat 2000; 121: 29-32.

24.- Sea la cul es la prevalencia de la trisoma 13? a) 1 / 20,000 b) 1 / 5,000 c) 1 / 2,500

d) 1 / 12,000 La trisoma 13, o sndrome de Patau, es una enfermedad congnita rara y grave; es causada por la presencia de un cromosoma extra en el par 13:1 su expresin puede ser 47, xx, + 13 47, xy, + 13.5. Fue inicialmente descrito en 1657 por Bartholin y en 1960 se clasifica como sndrome, Tras la descripcin de Patau.2 Se trata de un padecimiento raro cuya incidencia vara entre 1 x 5,000 y 1 x 12,000 nacidos vivos, representa el 1% de las causas de aborto espontneo1 y tiene ligero predominio en el sexo femenino 1:0.8. La oportunidad de haber tenido la experiencia de haber estudiado una nia recin nacida con esta anomala, es poco frecuente. Fuentes F. Sndrome de Patau (Trisoma 13); An Espaoles Pediat 2000; 121: 29-32.

25. Caso Clnico: Masculino de 25 aos que acude por esterilidad; es azoosprmico, tiene fenotipo eunucoide, ginecomastia, distribucin feminoide del vello pbico, testculos pequeos y cromatina sexual positiva en 8%. El diagnstico a considerar es? a) b) c) d) Sndrome de Noonan Sndrome de Turner Sndrome de Klinefelter Varn XX

Es la cromosomopata ms frecuente y la causa ms habitual de hipogonadismo hipergonadotrpico en el varn. Descrito en 1942 como un sndrome caracterizado por hipogonadismo, testes pequeos y duros, azoospermia y ginecomastia. Se comprob posteriormente que el cuadro corresponde a una patologa gentica, cuya alteracin cromosmica ms habitual es la presencia de un cromosoma X adicional, reflejando un cariotipo 47 XXY, que representa el 80% de los casos de SK, Rossodivita A, Colabucci F. Short stature in a patient with Klinefelter syndrome and growth hormone deficiency.Am J of Med Genet 1994; 49:244-6.

26. Cul es la prevalencia de la artritis reumatoide? a) 0.1 % b) 1 % c) 5 % d) 10 % Prevalencia de la enfermedad, 0.5 a 1%, con una relacin de mujer/ hombre del 2.5 al 1, mayor incidencia comprendida entre los 40 y 70 aos de edad.

Kirwan J. J Rheumatol. 1999.26:720-725. de

27. El siguiente antgeno del complejo principal de histocompatibilidad se asocia con la predisposicin a padecer artritis reumatoide. a) HLA-B27 b) HLA-DRw52 c) HLA-B8 d) HLA-DR4

Los aspectos genticos son determinantes en la artritis reumatoide, tanto en el inicio como en la perpetuacin de la enfermedad. Por ejemplo, se ha observado en personas de raza blanca una asociacin con varios subtipos del antgeno mayor de histocompatibilidad HDLDR4. Fox RI, Herrmann ML, Frangou CG, et al. Mechanism of action for leflunomide in rheumatoid arthritis. Clin Immunol 1999; 93(3): 198-208.

28. Indica cul es articulaciones son afectadas con mayor frecuencia en la artritis reumatoide? a) Metacarpo-falngicas b) Interfalngicas distales c) Codos d) Hombros

Es una enfermedad sistmica aunque las evidencias mayores son del sistema msculo esqueltico (articulaciones y estructuras relacionadas). La AR es el prototipo de las enfermedades reumticas articulares inflamatorias: es una poliartritis (polisinovitis) crnica generalmente simtrica, con cierta predileccin (al menos inicial) por las interfalngicas proximales de las manos, las metacarpofalngicas, muecas, rodillas y codos, aunque puede afectar cualquier articulacin que posea sinovial. PAC-MG 1, Parte D Libro 5, Cp., Artritis reumatoide Pg. 16 19. 29.- Cul es el sntoma que entra en los criterios de clasificacin de lupus eritematoso generalizado?

a) Alopecia b) Fotosensibilidad c) Artralgias d) Prdida de peso

Criterios de Clasificacin para el Diagnstico de Lupus Eritematoso Sistmico (LES) Erupcin malar: Eritema fijo, plano o alto, sobre las eminencias malares, que no suele afectar los surcos nasogenianos. Erupcin discoide: Placas eritematosas altas, con descamacin queratsica adherente y tapones foliculares; puede haber cicatrices atrficas en las lesiones ms antiguas. Fotosensibilidad: Erupcin cutnea a causa de una reaccin inslita a la luz solar, referida por el paciente u observada por el mdico. lceras bucales: Ulceracin nasofarngea, por lo comn indolora, observada por un mdico. Artritis: Artritis no erosiva que afecta dos o ms articulaciones perifricas, caracterizada por dolor a la palpacin, tumefaccin o derrame. Serositis: Pleuritis o pericarditis documentada por electrocardiograma o frote o evidencia de derrame pericrdico. Enfermedad renal: Proteinuria persistente mayor a 0,5g/da o 3+ o cilindros celulares. Trastorno neurolgico: Convulsiones o psicosis en ausencia de otra causa conocida. Trastorno hematolgico: Anemia hemoltica o leucopenia (< 4.000/mm3) o linfopenia: (< 1.500/mm3) o trombocitopenia (< 100.000/mm3) en ausencia de frmacos que produzcan esta alteracin. Trastorno inmunolgico: Anti-DNA, anti-Sm, y/o Anticuerpos antifosfolipdicos (AFL). Anticuerpo antinuclear: Un ttulo anormal de ANA por inmunofluorescencia o anlisis equivalente en cualquier momento y en ausencia de medicamentos relacionados con el sndrome de lupus de origen farmacolgico. Cualquier combinacin de 4 o ms de los 11 criterios, bien documentado durante cualquier intervalo de la historia del paciente, hace el diagnsticos de LES (especificidad y sensibilidad son del 95% y 75%, respectivamente). Petri M. Review of classification criteria for systemic lupus erythematosus. Rheum Dis Clin North Am. 2005 May;31(2):245-54.

30.- Selecciona los medicamentos considerados como la piedra angular del tratamiento de lupus eritematoso. a) Ciclofosfamida

b) Hidroxicloroquina c) Glucocorticoides d) Anitinflamatorios no esteroideos Corticoides Terapia inicial Los corticosteroides tienen efectos antiinflamatorios e inmunosupresores. Los esteroides se pueden aplicar por va tpica para el control de las Manifestaciones cutneas. A dosis bajas (menores de 15 mg de prednisona) se utilizan para el tratamiento de citopenias leves, pleuropericarditis y artritis que no responden al tratamiento con AINE y tambin como medicaciones puente para acompaar el perodo de latencia de la cloroquina (21) En manifestaciones que no ponen en peligro la vida (serositis, artritis, alteraciones hematolgicas, exantemas, etc.): dosis bajas, 0,5 mg/ kg/da de prednisona o equivalente, preferiblemente en una dosis matutina. En manifestaciones graves, pero que no ponen en Peligro la vida de forma inmediata (nefropata, afectacin neurolgica, anemia hemoltica): Dosis altas, 1-2 mg/kg/da en 1 a 3 dosis. En enfermedad grave con importante riesgo vital (Hemorragia pulmonar, afectacin neurolgica grave, insuficiencia renal rpidamente progresiva): 1) Bolos (pulsos) intravenosos de metilprednisolona: 20 mg/kg, hasta un mximo de 1 gramo, en 3 das consecutivos. American College of Rheumatology ad hoc committee on systemic lupus erythematosus guidelines. Guidelines for referral abd management of systemic lupus erythematosus in adults. Arthr Rheum 1999; 42:1785-1796.

31.- Qu rgano forma parte del sistema linfoide primario? a) Placas de Peyer b) Timo c) Bazo d) Ganglios linfticos

El timo es un rgano linfoide central que se origina del mesodermo. Este revestido por una capsula de tejido conectivo. Esta capsula penetra el parnquima tmico y lo divide en lbulos y lobulillos. La corteza representa la barrera hematotimica; es por ellos que se observa de color rojizo: por los capilares tipo I que se hallan en ella. Esta barrera no permite la entrada de linfocitos B al timo, lo que quiere decir que en este rgano solo hay linfocitos t. El timo se encuentra durante la niez, desarrolla durante la adolescencia en involuciona en edades avanzadas; su funcin es participar en la maduracin de los linfocitos T. RUTHERFORD R, Fifth edition, W.B Saunders Company Management of lymphatic disorders, seccin XX, 2000.

32.- Qu sustancia no pertenece a las inmunoglobulinas? a) IgG b) IgM c) IgC d) IgA Se concluye entonces, que no todos los anticuerpos son gammaglobulinas, por lo que Hebermans propone el trmino de inmunoglobulinas para designar a todas las sustancias con capacidad de anticuerpo. Hoy se conocen cinco tipos de inmunoglobulinas: IgM, IgA, IgG, IgD e IgE, cada una de ellas con ciertas caractersticas distintas. Galaktionov VG. (2004). Evolutionary development of the immunoglobulins super family. Izv Akad Nauk Ser Biol,, (2): 133-45.

33-. Antibitico recomendado para la diarrea del viajero: a) b) c) d) Ciprofloxacino Nitazoxanida Albendazol Gatifloxacino

PLAN DE ELECCION ALTERNATIVA ciprofloxacina 500 mg c/12 h v/o 3 a 5 das tratamiento emprico de diarrea aguda pefloxacina 400 mg c/12 h v/o 3 a 5 d norfloxacina 400 mg c/12 h v/o 5 das diarrea del viajero ciprofloxacina 500 mg c/12 h v/o 5 d salmonelosis menor shigelosis fluoroquinolona v/o 3 a 5 das norfloxacina 800 mg en TMP/SMX 160/800 mg c/12 h v/o 3 a 5d ampicilina TMP/SMX 160/800 mg c/12 h v/o 5 d TMP/SMX 160/800 mg c/12 h v/o 5 d norfloxacina 400 mg c/12 h v/o 3 a 5 d

dosis nica v/o ciprofloxacina 750 mg dosis nica v/o norfloxacina 400 mg v/o c/12 h 10 d salmonelosis mayor (tifoidea) ceftriaxona 2 a 4 g/d i/v 5 a 10 d diarrea post antibitico (Clostridium difficile) suspender el antibitico metronidazol 500 mg c/12 h v/o 10 d vancomicina 500 mg c/6 h v/o 10 d ciprofloxacina 500 mg v/o cloranfenicol 2 a 3 g/d por 21 das o 12 d despus de la apirexia c/12 h 10 d

Aun cuando la mayor parte de las diarreas de tipo secretor que se presentan en el viajero, son secundarias a E coli enterotoxignica, y el slo tratamiento con rehidratacin, subsalicilato de bismuto y antiperistlticos debera ser suficiente, se ha reportado que el uso de antibiticos con cobertura sobre estos microorganismos puede reducir la duracin de la sintomatologa de 3-4 das, a 24-26 horas. Lopez-Brea, Sanz Moreno J.C. Enteritis (bacterianas, vricas, parasitarias y fngicas) y toxiinfecciones alimentarias. Medicine. 1994;6(72): 3183-92.

34.- Un cuadro de diarrea con una duracin de ms de dos semanas, pero que generalmente no se extiende por ms all de cuatro se define como: a) b) c) d) Diarrea aguda Diarrea persistente Diarrea crnica Diarrea acuosa

La clasificacin de la diarrea de acuerdo al tiempo de duracin de este sntoma la divide en: aguda, con duracin de menos de 14 das, crnica con duracin de ms de 4 semanas, y persistente ha la que tiene una duracin menor del mes, y mayor a las dos semanas. Kasper DL, Braunwald E, Fauci AS, Hauser SL, Longo DL, Jameson JL. Harrisons Principles of Internal Medicine. McGraw Hill. 16 Ed. 225 p.

35.- De los siguientes antibiticos, en cul es indispensable ajustar la dosis de acuerdo a la funcin renal? a) Anfotericina B b) Dicloxacilina c) Metronidazol d) Ofloxacina

Aun cuando algunos de los antibiticos mencionados tienen nefrotoxicidad (Anfotericina) o pertenecen a grupos en los que otros compuestos requieren de ajuste a la funcin renal, como los betalactmicos (dicloxacilina), en este grupo slo la ofloxacina, parte de las quinolonas, requiere de ajuste para la funcin renal.

Kasper DL, Braunwald E, Fauci AS, Hauser SL, Longo DL, Jameson JL. Harrisons Principles of Internal Medicine. McGraw Hill. 16 Ed. 789-806 pp.

36.- Le interconsultan de la UTI para valorar un paciente con sepsis abdominal. Lleva dos das con tratamiento a base de ceftriaxona, metronidazol y amikacina, el cual provee de una cobertura conveniente para los microorganismos probablemente involucrados. Despus de revisar al paciente que ha tenido una evolucin favorable, usted sugiere que se aplique el aminoglucsido en monodosis ya que: a) b) c) d) De esta forma aumenta su potencia. Como tiene un efecto post antibitico corto su efecto es mejor Disminuyen sus efectos adversos Hace sinergismo con el Metronidazol

Una caracterstica farmacolgica de los aminoglucsidos es su actividad dependiente de concentracin, por lo que su aplicacin en bolos, con lo que se logran niveles pico ms altos, es de mayor conveniencia para aprovechar este fenmeno. Adems, el importante efecto postantibitico que tiene, permiten que la actividad dure ms all del tiempo en que sus concentraciones se encuentran por arriba de la mnima inhibitoria, favoreciendo una posologa cmoda cada 24 horas. Por otra parte, se ha visto que los efectos adversos (nefrotoxicidad y ototoxicidad), se encuentran ms relacionados con los niveles valle. Estos se mantienen a niveles bajos cuando se utiliza el esquema en monodosis, disminuyendo la probabilidad de la presentacin de estos efectos adversos. Kasper DL, Braunwald E, Fauci AS, Hauser SL, Longo DL, Jameson JL. Harrisons Principles of Internal Medicine. McGraw Hill. 16 Ed. 789-806 pp.

37.- Acude a consulta una mujer de 25 aos, cursando su 14 semana de gestacin, por tenesmo vesical, disuria y escalofro. Por su estado actual, cul de los siguientes antimicrobianos recomendara? a) b) c) d) Ampicilina Metronidazol Tetraciclina Levofloxacina

El Metronidazol no ha mostrado efectos txicos en humanos, pero es teratognico en modelo animal. Las tetraciclinas ocasionan coloracin anormal de los dientes, hepatotoxicidad y alteracin en el desarrollo de huesos. Las sulfas podran tener un efecto deletreo en el primer trimestre dada su actividad como antimetabolitos, y en los ltimos meses pueden favorecer kernicterus en el recin nacido si es que tiene alteraciones metablicas que favorezcan anemia hemoltica. Las quinolonas se han asociado a malformaciones seas en modelos animales, y se recomienda evitarlas si existen mejores opciones. Kasper DL, Braunwald E, Fauci AS, Hauser SL, Longo DL, Jameson JL. Harrisons Principles of Internal Medicine. McGraw Hill. 16 Ed. 789-806 pp.

38.- El efecto postantibitico se refiere a: a) Las reacciones adversas despus de administrar un antimicrobiano b) Las concentraciones sricas pico c) La supresin persistente del crecimiento bacteriano posterior a la exposicin a un antimicrobiano d) Las pruebas de susceptibilidad antimicrobiana

Algunos antibiticos mantienen supresin del crecimiento bacteriano aun despus de que sus niveles disminuyen por debajo de la concentracin inhibitoria mnima. Este efecto permite que se apliquen en posologas en que el tiempo entre una dosis y otra es relativamente largo, como en los aminoglucsidos y las quinolonas. Kasper DL, Braunwald E, Fauci AS, Hauser SL, Longo DL, Jameson JL. Harrisons Principles of Internal Medicine. McGraw Hill. 16 Ed. 789-806 pp.

39.- Diagnostic un absceso heptico amibiano en un paciente masculino de 24 aos. Indic tratamiento con Metronidazol, con lo que se observ respuesta clnica favorable. Posteriormente complementa el tratamiento con el siguiente frmaco: a) Emetina b) Cloroquina

c) Albendazol d) Iodoquinol (hidroxiquinolena) l Posterior al tratamiento para la forma invasiva (trofozoito), para el cual el tratamiento ms efectivo al momento es el Metronidazol, pero sobre el que tienen tambin efectos la emetina, Cloroquina y tinidazol, se recomienda utilizar un agente con actividad cisticida (actividad luminal), como el iodoquinol o la Paromomicina. Kasper DL, Braunwald E, Fauci AS, Hauser SL, Longo DL, Jameson JL. Harrisons Principles of Internal Medicine. McGraw Hill. 16 Ed. 1214-1217 pp.

40.- Paciente de 52 aos de edad con diagnstico de neumona adquirida en la comunidad, quien recibi tratamiento antibitico a base de Ceftriaxona. No tuvo respuesta adecuada, y en el estudio diagnstico para determinar la causa, se encontr un derrame pleural del 60%, el cual se puncion. En el estudio citoqumico y bacteriolgico de este lquido, se reportan cocos grampositivos en la tincin de Gram, y un pH de 7. Con esto, usted considera como indispensable: a) b) c) d) Iniciar cobertura con vancomicina. Colocacin de sonda endopleural. Decorticacin. Intubacin orotraqueal y ventilacin con volmenes altos.

Los hallazgos del citoqumico y tincin son caractersticos de empiema. El drenaje del mismo es la maniobra teraputica base para el tratamiento y resolucin del mismo. Kasper DL, Braunwald E, Fauci AS, Hauser SL, Longo DL, Jameson JL. Harrisons Principles of Internal Medicine. McGraw Hill. 16 Ed. 1536 p.

41.- Hombre de 37 aos de edad, diseador grfico, homosexual, con infeccin por VIH diagnosticada hace 2 aos, acude al servicio de urgencias por presentar fiebre de 15 das de evolucin, ataque al estado general, prdida de peso, disnea progresiva y tos productiva. Se realiz tincin de la expectoracin, que report bacilos cido-alcohol resistente. Su cuenta de CD4 realizada hace 1 mes fue de 100 cl/mL y una carga viral reciente fue de 250,000 copias. En qu estadio CDC se encuentra el paciente? a) b) c) d) A3 B3 C3 B2

El estadio C3 est definido por una cuenta de CD4 menor de 200 cl/mL y la presencia de una condicin clnica indicativa de Sndrome de Inmunodeficiencia Adquirida, entre las que se encuentra la infeccin por Mycobacterium tuberculosis en cualquier variedad. Kasper DL, Braunwald E, Fauci AS, Hauser SL, Longo DL, Jameson JL. Harrisons Principles of Internal Medicine. McGraw Hill. 16 Ed. 1076 p.

42.- La prueba de tamiz de glucosa para la deteccin de diabetes gestacional se debe de realizar? a) b) c) d) 4 a 8 semanas de embarazo. 12 a 14 semanas de embarazo. 24 a 28 semanas de embarazo. 34 a 38 semanas de embarazo.

Est indicado en la mujer embarazada con factores de riesgo, a las 24-28 semanas de gestacin. Actualmente se recomienda no realizar el cribado en la mujer de bajo riesgo (edad inferior a 25 aos, normopeso, no antecedentes familiares de diabetes, no pertenencia a grupo tnico con alta prevelencia de diabetes). American Diabetes Association. Preconception Care of Women with Diabetes. Position Statements.Diabetes Care 1999; 20 (suppl 1): s62-s65.

43.- Clula involucrada en el origen (patogenia) de la atopia: a).- neutrfilo b).- Eosinfilo c).- Linfocito CD4+ Th1 d).- Linfocitos CD4+ Th2 Los linfocitos en el infiltrado del eczema atpico son, predominantemente, de tipo adyuvante CD4+, con un ndice medio CD4:CD8 de 7:1. Los linfocitos CD4+ son mayoritariamente, de tipo memoria y expresan elevados niveles de la lectina Sialyl lewisx/e ligando de la molcula de adhesin leucocitaria endoteial-1 /ELAM-1). Se ha demostrado ICAM-1 sobre los queratinocitos epidrrmicos focalmente, situados cerca del infiltrado linfocitario. Estos queratinocitos liberan interleucina (IL)-1 y 8. En el eczema atpico las clulas dendrticas epidrmicas y drmicas expresan el mismo fenotipo, son capaces de expresar un receptor especfico para la IgE y de activar los linfocitos T del mismo individuo en ausencia de antgenos. Todas estas caractersticas sugieren la existencia de un proceso de presentacin antignica. Clinical Aspects of Immunology Third Edition

P.G.H. Gell, R.R.A. Coombs and P.J. Lachmann Blackwell Scientific Publications 1975

44.- Principal mediador qumico responsable del broncoespasmo en el paciente con asma: a).- Histamina b).- Serotonina c).- PGE2 d).- LTC4, D4, E4 En la membrana del eosinfilo se liberan derivados del cido araquidnico, producto de su metabolismo por la va lipoxigenasa que lleva a la produccin de Leucotrienos (LTD4 y LTC4), los sulfapptidos LTS y LTC4 y sus derivados LTD4 y LTE4 los cuales provocan contraccin muscular lisa potente y tiene propiedades vasoactivas, se describen ms de quince productos derivados de la va lipoxigenasa a travs del eosinfilo. Existen adems en la membrana celular del mastocito enzimas como la Adenilciclasa y Guanilciclasa que catalizan la formacin del AMPc y el GMPc los cuales influyen en diferentes funciones entre las que figuran la liberacin de mediadores para las clulas sensibilizadas y el proceso contrctil del msculo liso bronquial. La liberacin de mediadores inducida por alergenos es potenciada por agentes colinrgicos y la prostaglandina F2 al aumentar el GMPc y por estimulacin adrenrgica que disminuye el AMPc o posiblemente por bloqueo de los receptores de adenosina. En el paciente asmtico hay desbalance entre el ATP y el GMP que favorece la liberacin de mediadores qumicos.10

MAZZEI J. A Asma en el segundo milenio MEDICINA - Volumen 60 - (Supl I), 2000 pp.5-10

45.- Principal causa de rinitis alrgica perenne: a).-caros en polvo casero b).- Plenes c).- Cucarachas d).- Caspa

La Rinitis Alrgica se divide en estacional y perenne, sta divisin es causada por la presencia del alergeno en el aire, as, si slo se da la presencia de ste en una o dos estaciones del ao, por ejemplo los plenes, se denomina RA estacional, pero si el alergeno se encuentra en el aire todo el ao, por ejemplo en el polvo casero con predominancia de caros, se llamar perenne. Tambin puede ocurrir que un individuo tenga alergia a ms de un alergeno, los cuales tengan presencia estacional pero en diferentes estaciones, que pueden ser continuas o no, entonces a sta presentacin se denomina RA estacional dual (33,34). En la bibliografa mundial se reporta que el porcentaje de pacientes que sufren RA perenne es igual al 40 a 50% de personas que tienen RA (4,7).

International Rhinitis Management Working Group. International Consensus Report on the Diagnosis and Management of Rhinitis. Eur J Allergy Clin Inmunol 1994; 49 (suppl). 234-36. 46.- El estudio de eleccin para hacer diagnstico de esofagitis es: a) b) c) d) La serie Gastro Duodenal Ph metria Endoscopia Impedanciometria endolumin.

Las tcnicas de cromo endoscopia se han postulado como alternativas para disminuir los errores diagnsticos. La doctora Irene Canto, en un estudio realizado (14) en 26 pacientes con histologa de esfago de Barrett, mostr para la tincin de azul de metileno una sensibilidad del 95% y una especificidad del 97%, con valores predictivos positivo y negativo del 98 y 92%, respectivamente. Por el contrario, nuestro estudio, que incluy una muestra de 105 pacientes enrolados al azar, mostr que la sensibilidad fue de 55% y la especificidad fue del 63,6%, con valores predictivos positivo y negativo del 23 y 87%, respectivamente. Esto parecera desvirtuar el valor de esta tcnica en el contexto del trabajo cotidiano de una unidad de endoscopia, por el alto nmero de falsos positivos, y an de falsos negativos, con datos en la literatura que demuestran que hay focos de metaplasia an en la mucosa de aspecto normal Bibliografa: Urgencias en Pediatra, Interamericana.McGraw Hill. Captulo: Urgencias Mdico Quirrgicas, Seccin XXIII, Pg. 182-194 Enciclopedia Mdico Quirrgica-Pediatra 4-014-L-10-2005

47.- Cul es la complicacin mas grave de la Enfermedad de Hirschsprung? a) b) c) d) Vlvulus Invaginacin Intestinal Sangrado de Tubo Digestivo Enterocolitis

Los problemas con un nio que padece la enfermedad de Hirschsprung dependen de la porcin de intestino que tiene clulas nerviosas normales. El setenta por ciento de los bebs con la enfermedad de Hirschsprung carecen de clulas nerviosas solamente en una porcin de los ltimos treinta o sesenta centmetros (uno o dos pies) del intestino grueso. Siendo una de las complicaciones mas severas la enterocolitis Operative Pediatric Surgery. Moritz M. Ziegler. International Edition, pg 445 463

Bibliografa: Urgencias en Pediatra, Interamericana.McGraw Hill. Captulo: Urgencias Mdico Quirrgicas, Seccin XXIII, Pg. 182-194 Enciclopedia Mdico Quirrgica-Pediatra 4-014-L-10-2005

48.- La prueba de Coombs directa detecta: a).-Ac incompletos acoplados a los eritrocitos b).- Ac completos acoplados a los eritrocitos c).- Ac incompletos circulantes d).- Ac completos circulantes Si la prueba de Coomb's directa resulta positiva es indicativo de que existen anticuerpos unidos a los hemates. Normalmente, cuanto ms intensa sea la reaccin (ms positiva sea la prueba de Coomb's), habr mas cantidad de anticuerpo unido a los hemates. Esta prueba detecta la presencia de anticuerpos pero no indica sin embargo, el origen o el tipo exacto de anticuerpo. Puede obedecer a una reaccin transfusional, a una reaccin de tipo autoinmune, a una infeccin, a frmacos, a una incompatibilidad Rh madre-hijo, o ms raramente, a una exposicin al fro. Cabe destacar que un porcentaje pequeo de la poblacin sana presenta tambin una reaccin positiva a la prueba de Coomb's directa sin presentar por ello anemia hemoltica.

Rakel P, ed. Conns Current Therapy 2005. 57th ed. Philadelphia, Pa: WB Saunders; 2005: 471-473.

49.- Cursa con PANCA (Ac anti-citoplasma de neutrfilos anti-mieloperoxidasa): a).- Poliarteritis nodosa clsica b).-Poliarteritis nodosa microscpica c).- Alveolitis alrgica extrnseca d).- Granulomatosis de Wegener Datos de laboratorio: El aumento de los reactantes de fase aguda suele ser menos intenso que en la PAN. (Poliarteritis Nodosa)Puede haber anemia moderada o intensa en el caso de existir hemorragia pulmonar o digestiva. Lo ms caracterstico de la PA (Poliasrteritis Nodosa ). Es la presencia de anticuerpos anticitoplasma de los neutrfilos (ANCA) con patrn perifrico en inmunosfluorescencia (p-ANCA) y que por ELISA (enzyme-linked immunosorbent assay) corresponden A anticuerpos dirigidos frente a la mieloperoxidasa (anti-MPO) 11,12. Petty RE, Cassidy JT, Polyarteritis nodosa and related vasculitides.

En: Cassidy JT, Petty RE (eds.) Textbook of Pediatric Rheumatology. 4 edicion, Philadelphia: Saunders Company. 2001:595-603. 50.- En un frotis vaginal la presencia al microscopio de clulas clave son propias de la infeccin por: a) Cndida albicans b) Gardnerella c) Tricomonas d) Gonococos

La infeccin por gardnerella (bacteria) tiende a producir una secrecin blanca, gris o de color amarillo turbio, con un olor ftido o a "pescado" que aumenta cuando la secrecin se vuelve alcalina, como sucede despus del coito o de lavarse con jabn. Puede haber prurito o irritacin vulvar, pero por lo general no son muy pronunciados. Al microscopio se aprecia las famosas clulas clave. Realizando el diagnostico diferencial.

Pernoll M. Enfermedades de transmisin sexual. En: Manual de Obstetricia y Ginecologa. Benson/Pernoll Editores.Editorial Interamericana S A. Mxico DF, 1994

51.- Complicacin que puede surgir por la enfermedad o por el tratamiento en casos de tuberculosis urinaria: a) Gangrena gaseosa b) Divertculos ureterales c) Cncer de urotelio d) Estenosis ureteral Durante la fase de diseminacin hematgena (primoinfeccin) se produce siembra de bacilos en ambos riones en el 90% de los casos, sin embargo, la enfermedad clnica generalmente es unilateral. El perodo de latencia entre la siembra y la enfermedad clnica oscila entre 10 y 40 aos, afectando principalmente a pacientes por debajo de los 50 aos. La lesin inicial microscpica se localiza en los glomrulos en forma de granulomas microscpicos. Al avanzar la enfermedad se produce afectacin ms distal hasta la aparicin de una papilitis necrotizante, momento en el cual puede ya existir paso de bacilos a la va excretora, donde por procesos inflamatorios ocasionar estenosis a nivel de los infundbulos caliciales, pelvis y urter, con hidronefrosis secundaria.

Manual CTO 7 edicin, Urologa, pg5

52.- Forma ms frecuente de la infeccin por papiloma virus en varones: a) Verrugas plantares b) Verrugas larngeas c) Subclnica en genitales d) Condilomas prepuciales En diferentes estudios de varones compaeros de mujeres con lesiones cervicales por virus del papiloma humano se demostr que el 88% tenan signos histolgicos de condilomas, de los cuales el 72% eran en forma subclnica es decir no haba ningn tipo de lesin visible o sntoma. Otros estudios han demostrado que hasta el 66% o mas de los compaeros de mujeres con neoplasias intraepiteliales cervicales tienen infecciones subclnicas por virus del papiloma humano en el pene. Obstet Gynecol 2001;185:308-17. Kurtycz DFI, Hoerl HD. Thin-Layer Technology: Tempered Enthusiasm. Diag Cytopath 2000;23(1):1-5.

53.- El clsico inicio del dolor por apendicitis es en: a. b. c. d. Hipogastrio Fosa lumbar derecha Epigastrio Fosa iliaca derecha

Cuadro clnico y diagnstico. El cuadro clnico suele instaurarse en pocas horas. La secuencia clsica de los sntomas es la siguiente: 1 dolor abdominal en epigastrio;2 nuseas, vmitos; 3 cambio del dolor hacia fosa ilaca derecha y aparicin de hiperestesia a la presin profunda; 4 fiebre 38.3-39.4C; 5 leucocitosis (10000-20000/dL). Cuando el orden de aparicin de estos sntomas es diferente debe reconsiderarse el diagnstico. El dolor es el sntoma ms constante. En forma caracterstica el dolor aparece por la noche, en general en el epigastrio o la regin periumbilical; otras veces es difuso en todo el abdomen o, con menor frecuencia, se localiza directamente en la fosa ilaca derecha. Es un dolor continuo, de intensidad moderada, que aumenta con la tos, los movimientos respiratorios o la deambulacin y no cede tras el vmito. Puede tener exacerbaciones clicas y obliga a guardar

cama. A las pocas horas se localiza definitivamente en la fosa ilaca derecha y en algunos pocos casos puede presentar irradiacin testicular. El origen del dolor es la distensin de la luz apendicular. Una mejora brusca del dolor suele indicar perforacin del rgano. Al palpar el abdomen se aprecia dolor a la presin profunda y puede observarse ya rigidez refleja de la pared abdominal en la fosa ilaca derecha. No obstante, la aparicin de rigidez parietal, inconstante, demuestra ya la existencia de irritacin del peritoneo parietal (peritonitis localizada), con lesiones apendiculares avanzadas. Ferrada R.: Guas para el manejo de urgencias- Apendicitis aguda. Hospital universitario Evaristo Garca. 1996. Guss D.: Comparison of men and women presenting with acute appendicitis. Am. J. Emer. Med. 2000. 18: 4: 372-376. 54.- Las siguientes son manifestaciones del sndrome de HELLP: a) Hemlisis , elevacin de enzimas hepticas y plaquetopenia, b) Hiperglicemia, hiperkalemia, disminucin de transaminasas. c) Hemlisis. d) Insuficiencia renal aguda.

Los criterios para diagnosticar el Sndrome HELLP, son los que utiliz Sibai para clasificar a esta entidad y se basa en: Hemlisis: - esquistocitos en el frotis de sangre perifrica Bilirrubina: - mayor o igual a 1,2 ml/dl Hepatoglobinas: ausentes en plasma Enzimas hepticas: elevadas (GOT mayor 72 UI/l y LDH mayor a 600 UI/l). Plaquetas: menor 100X103/mm2.

Nassif J. Actualizacin de la atencin en hipertensin y embarazo. Bs. As, Argentina. Revista de Obstetricia y Ginecologa de la Provincia de Buenos Aires; 1998: volumen 29 n 157.

55.- Cul es la complicacin temprana ms frecuente del sarampin?:

a) b) c) d)

Neumona. Otitis media aguda. Encefalitis. Laringotraqueitis.

La complicacin temprana ms frecuente es la otitis media aguda siendo del 7 al 9 %, neumona del 1 al 6 %, encefalitis 1:1,000 casos, panecencefalitis esclerosante subaguda 1:100,000 casos y neumona de clulas gigantes es muy rara. Gonzlez-Saldaa N, Infectologa Clnica Peditrica, 7 edicin, pginas 353-362. 56.- En el sarampin la complicacin que se asocia a mayor mortalidad es? a) b) c) d) Neumona. Otitis media aguda. Encefalitis. Laringotraquetis

La neumona como complicacin representa del 60 al 70 % de las defunciones por sarampin. Puede originarse por la propia infeccin viral o por una infeccin bacteriana sobreagregada (Streptococcus pneumoniae, Staphylococcus aureus, Haemophilus influenzae). Gonzlez-Saldaa N, Infectologa Clnica Peditrica, 7 edicin, pginas 353-362.

57.- Cual de los marcadores sricos para diagnosticar Infarto Agudo al Miocardio traduce lesin celular (necrosis)? a) Creatinfosfokinasa. b) CPK-MB c) De4shidrogenasa lctica d) Troponina I Las troponinas cardiacas son protenas que forman parte de los mecanismos de regulacin de la contraccin del msculo cardiaco, estn presentes en las fibras miocrdicas. La troponina es una protena globular de gran tamao, contiene tres subunidades polipeptdicas: troponina C (fijadora de calcio), troponina I (inhibidora de la interaccin actina-miosina) y troponina T (fijadora de tropomiosina). Cuando se necrozan las clulas del tejido miocrdico pierden la integridad de la membrana celular y las molculas intracelulares difunden hacia la microcirculacin y a los linfticos. Estas macromolculas se detectan en la circulacin perifrica y constituyen los marcadores bioqumicos especficos de dao al miocardio. .

-Owen Avril. Cardiac troponins: improved diagnosis and cost benefits. 2001. Clinical Laboratory International. Volume 25. No. 8. P.14-15.

58.- Algunas de las causas de abdomen agudo en el recin nacido son?

a) b) c) d)

Atresia intestinal, malrotacin, enterocolitis. Enterocolitis, invaginacin intestinal, apendicitis. leo meconial, apendicitis, complicaciones de la gastroenteritis. Enfermedad de Hirchsprung, malformaciones anorrectales, oclusin por scaris.

Origen digestivo:

Recin nacidos. A esta edad las causas Ms comunes van a estar en relacin con Malformaciones del aparato digestivo o extradigestivas, pero ms raramente la Causa sera de carcter mdico.

Malrotacin y vlvulo intestinal. Atresia o bandas duodenales. Atresia yeyunoileal. leo o tapn meconial. E. de Hirschprung. Colon izquierdo hipoplsico. Obstruccin funcional. Adinamia Congnita. Ectopia-atresia anal. Duplicaciones Intestinales.

Origen extradigestivo:

Onfalocele. Extrofa vesical. Hernia diafragmtica.

Nelson N. Dolor abdominal. Tratado de Pediatra (Ed.15). Mc Graw-Hill, Interamericana, Vol. 1:1303-6.

59.- La atresia de esfago ms frecuente es? a) b) c) d) Atresia de esfago sin fstula Atresia de esfago con fstula en cabo proximal Atresia de esfago tipo H Atresia de esfago con fstula en cabo inferior.

Es una malformacin relativamente frecuente, presentndose entre 1:2500 nacimientos, sin predominio sexual y con cierta incidencia familiar. FORMAS ANATMICAS DE LA ATRESIA DEL ESOFAGO

TipoI: AE completa sin fstula (8%), Tipo. II: AE con fstula traqueoesofgica proximal (1%). Tipo III: AE con fistula traqueoesofgica distal (80%). Tipo IV: AE con fstula traqueoesofgica y distal (1%). Tipo V: Fstula traqueoesofgica sin AE (4%). Bibliografa: Urgencias en Pediatra, Interamericana.McGraw Hill. Captulo: Urgencias Mdico Quirrgicas, Seccin XXIII, pg. 774-778. Operative Pediatric Surgery. Moritz M. Ziegler. International Edition, pg. 349- 354. Ciruga Peditrica, Ashcraft - Holder Interamericana.McGraw Hill pg. 257 a 277

60.- El lupus eritematoso generalizado es ms frecuente en: a) La adolescencia b) Mujeres en edad reproductiva c) La infancia y personas mayores de 60 aos d) Personas mayores de 60 aos

EPIDEMIOLOGA El LES es una enfermedad de distribucin mundial, afecta a todas las razas aunque es de mayor gravedad en la raza negra, tiene predominio por el sexo femenino en la proporcin de 9:1, se manifiesta en cualquier edad siendo ms frecuente en la etapa productiva y reproductiva de la vida (entre 20 y 40 aos). GUTIRREZ C, MOZO L. Diagnstico inmunolgico: enfermedades auto inmunes. En: Inmunologa Clnica. Bases moleculares y celulares. J. Pea, ed. Madrid. Arn. 107-118, 1996.

61.- Las siguientes manifestaciones clnicas son las ms frecuentes en el lupus: a) Las cutneas y las articulares b) Las cardacas c) Las pulmonares d) Las musculares

Despus de la fatiga, la fiebre, la astenia, las alteraciones de la piel, el pelo y las mucosas ocupan el segundo lugar entre las manifestaciones clnicas (85% de los casos). El clsico eritema facial en alas de mariposa ocurre en 52% y es consecuencia frecuente de fotosensibilidad; tambin se localiza en el trax, espalda y brazos como lesiones eritematosas simtricas superficiales con zonas centrales atrficas anulares.

El lupus eritematoso discoide se localiza en la piel cabelluda, pabellones auriculares, cara y cuello y se asocia con frecuencia a fotosensibilidad y fenmeno de Raynaud. GUTIRREZ C, MOZO L. Diagnstico inmunolgico: enfermedades auto inmunes. En: Inmunologa Clnica. Bases moleculares y celulares. J. Pea, ed. Madrid. Arn. 107-118, 1996.

62.- El estadio 2 de la nefropata diabtica se caracteriza por aparicin de: a) Hipertensin arterial sistemica b) Microalbuminuria c) Eritrocituria d) Proteinuria Clasificacin de nefropata diabtica.

NEFROPATIA DIABETICA
Estadio I: Hiperfiltracin. Estadio II: Microalbuminuria (30 a 300 mg/d). 6 a 15 aos. Estadio III: Proteinuria. Hipertensin. Perdida de GFR 10 ml/min al ao. Estadio IV: Insuficiencia renal.

Uso exclusivo Universidad La Salle , Dr. Cano

Report of the Expert Committee on the Diagnosis and Classification of Diabetes Mellitus. Diabetes Care. 1997;20:1183-97.

63.- La metas de control glucmico en el paciente diabtico adulto son: a) b) c) d) Glucosa de ayuno < 180, 2 horas postprandial < 240, HbA1c <8 Glucosa de ayuno: 140-180, 2 horas postprandial < 200, HbA1c <7 Glucosa de ayuno: 70-100, 2 horas postprandial < 140, HbA1c < 6 Glucosa de ayuno: 90-130, 2 horas postprandial < 180, HbA1c < 7

Para definir con claridad los objetivos del tratamiento de la hiperglucemia, los expertos recomiendan alcanzar y mantener rangos no diabticos, es decir, una glucemia en ayunas inferior a 130 mg/dL, junto con glucemias postprandiales menores de 180 mg/dL (idealmente menores a 140 mg/dL) y cifras de hemoglobina glucosilada (HbA1c) menores a 7%. Report of the Expert Committee on the Diagnosis and Classification of Diabetes Mellitus. Diabetes Care. 1997;20:1183-97.

Sheperd PR, Kahn BB. Mechanisms of disease: Glucose transporters and insulin action. Implications for insulin resistance and diabetes mellitus. N Engl J Med, 1999;341:248257. Bloomgarden ZT. Obesity and diabetes. Diabetes Care. 2000;23:1584-1590. Belfiore F, Lannello S, Camuto M, Fagone S, Calaveri A. Insulin sensitivity of blood glucose versus insulin sensitivity of blood free acids in normal, obese, and obesediabetic subjects. Metabolism. 2001;50:573-582.

64.- Cul de las siguientes vacunas son conjugadas?

a) Pneumococo de 7 serotipos, meningococo de cuatro serotipos y Haemophilus influenzae tipo B. b) Sabn, triple viral y varicela. c) Influenza, hepatitis A y hepatitis B. d) BCG, pneumococo polivalente y DPT. Conjugadas de Haemophilus influenzae tipo b (Hib) Las vacunas conjugadas de Hib unen covalentemente el polisacrido de Hib con una protena acarreadora, que induce una respuesta inmunolgica dependiente de clulas T y memoria inmunolgica que no se induce con el polisacrido solo en la vacuna. Las protenas acarreadoras autorizadas para infantes incluyen el toxoide tetnico (vacuna PRP-T); un mutante, toxina diftrica no txica (HbOC); y la protena de membrana externa de Neisseria meningitidis (PRP-OMP). Una cuarta vacuna (PRP-D), conjugada al toxoide diftrico, est autorizada slo para uso en nios mayores (15 meses de edad o mayores) debido a su inmunogenicidad ms baja en infantes.

1. Ammann AJ, Ashman RF, Buckley RH et al. Use of intravenous gammaglobulin in antibody immunodeficiency: Results of a multicenter controlled trial. Clin Immunol Immunopathol 1982; 22:60-7. 2. Roifman CM, Lederman HM, Lavis S, Stein LD, Levison H, Gelfand EW. Benefit of intravenous IGG replacement in hypogammaglobulinemic patients with chronic sinopulmonary disease. Am J Med 1985; 79:171-4. 3. American Academy of Pediatrics, 1994, Red Book: Report of the Committee on infectious Diseases, 23rd ed. Elk Grove Village, IL, 1994.

65.- Qu eventos adversos podemos esperar por la aplicacin de Difteria Pertusis Ttanos? a) Adenitis supuras y encefalitis. b) Parlisis flcida aguda. c) Episodio hipotnico e hiporreactivo y llanto incontrolable y persistente por ms de 3 horas. d) Exantema maculopapular y fiebre.

Los eventos sistmicos se presentan dentro de las 48 horas despus de la vacunacin, se han notificado (fiebre en el 40% de los vacunados, en el 5 % llanto persistente e incontrolable por ms de tres horas, somnolencia, irritabilidad y malestar general. En menos del 3% cefalea, convulsiones, calosfro, mialgias y artralgias. Committee on Infectious Diseases 2006. Red Book 2006, 27th Ed. American Academy of Pediatrics; 2006

66.- En un paciente del sexo masculino de 35 aos de edad con una dermatosis diseminada a codos y rodillas con placas eritemato-escamosas usted har el diagnstico de ? a) Psoriasis b) Dermatitis atpica c) Dermatitis seborreica d) Dermatitis de contacto La psoriasis es una enfermedad crnica que evoluciona en brotes de causa desconocida, que se caracteriza por placas eritemato escamosas en diferentes partes de la piel. Se presenta por igual en hombres y en mujeres, en todas las edades, predominando en jvenes y ms frecuentemente en personas de piel blanca (parece que la presencia de melanina protege contra la enfermedad. Tambin es ampliamente reconocido que la psoriasis es una enfermedad familiar y hereditaria (ditesis psorisica). Los sitios de predileccin para que aparezcan las lesiones, son los salientes seos como codos, rodillas, y la piel cabelluda as como la regin sacrocoxgea. Christophers E, Krueger G G. Psoriasis. En : Fitzpatrick TB, EISEN AZ, Wolff K. Dermatologa en Medicina General Buenos Aires: Editorial Panamericana 1988. P. 585-591

Christophers E, Schubert C, Schrder J M. Psoriasis. Dermatologa. 1992; 45.

67.- En este caso usted buscar el siguiente signo cutneo? a) Signo de Koebner b) Signo de Asboe c) Signo del roco sangrante d) Signo de la ua Signos cutneos en psoriasis. Signo de Auspitz o signo del roco sangrante: consiste en un piqueteado hemorrgico que aparece tras el desprendimiento de la membrana de Duncan-Dulckley; este signo aparece como consecuencia del dao vascular de los vasos de las papilas drmicas al desprender la epidermis adelgazada suprayacente a las papilas. Christophers E, Krueger G G. Psoriasis. En : Fitzpatrick TB, EISEN AZ, Wolff K. Dermatologa en Medicina General Buenos Aires: Editorial Panamericana 1988. P. 585591 Christophers E, Schubert C, Schrder J M. Psoriasis. Dermatologa. 1992; 45.

68.- El tratamiento de eleccin es: a) Queratolticos como: pomada salicilada b) Emolientes como: cold cream c) Secantes como: agua de alibour d) Antifngicos como: ketoconazol

Tratamiento tpico: Queratolticos (cido saliclico, urea, cido lctico). Se emplean para eliminar el exceso de escamas fuertemente adheridas y sobre todo en las hiperqueratosis palmo-plantar. Las curas oclusivas, que son tiles para hidratar, facilitan la penetracin percutnea de algunos medicamentos e inhibe la actividad mittica de las clulas epidrmicas. Estas curas oclusivas no deben prolongarse ms de 6-8 horas para evitar as la aparicin de maceracin e infeccin

Burton JL, Rook A, Wilkinson DS. Eczema, lichen simple, erythroderma and pruriga. En: Rook a, Wilkinsons DS, Ebling FJG. Texbook of Dermalogy 14 th. Ed. Boston: Black Well 1986. pg. 985-1003 Brody I J. Invest. Derm. 1984. 82: 460

69.- Qu tipo de diseo de investigacin son los estudios de casos y controles? a) b) c) d) Observacional descriptivos Experimental descriptivo Observacional analtico

Ensayo clnico

Los estudios descriptivos se relacionan con la distribucin de las enfermedades, incluyendo la consideracin de qu poblacin o subgrupos desarrollan o no enfermedad, en que localizacin geogrfica es ms o menos comn, y cmo la frecuencia de ocurrencia vara en el tiempo. Esto incluso puede llegar a dirigir a la formulacin de una hiptesis epidemiolgica que sea consistente con el conocimiento existente de la ocurrencia de enfermedad. Los estudios analticos que se focalizan en los determinantes de una enfermedad prueban las hiptesis formuladas desde los estudios descriptivos, con el objetivo final de juzgar si una exposicin particular se asocia, causa o previene enfermedad, amplindose tambin a aquellos estudios que evalan los factores que se relacionan con distintos resultados de una enfermedad (ej: mejora o muerte).

CARACTERSTICAS DE UN ESTUDIO ANALTICO: Existe un grupo control Comparabilidad en los grupos

Control de factores diferentes al de inters Medicin de efecto del azar (pruebas de significacin) Dentro de los estudios analticos estn: Estudios de Casos y Controles Estudios de Cohorte Ensayos ClnicosRuiz A, Morillo L. Epidemiologa Clnica. Investigacin clnica aplicada. Bogot, Editorial Mdica Panamericana, 2004. - Fletcher R, Fletcher S, Wagner E. Epidemiologa Clnica: aspectos fundamentales MASSONWilliams & Wilkins Espaa, S.A. 2 edicin en espaol, 1998. - Hulley SB, Feigal D, Martin M, Diseo de Investigacin clnica. Ediciones Doyma, 1997

70.- Estamos ante una epidemia cuando? a) La tasa de ataque de la enfermedad sea mayor de 10 por 1000 habitantes b) Los casos de una enfermedad aumentan en exceso con respecto a la expectativa normal para una poblacin en ese momento c) Una enfermedad tiene una tasa baja de aparicin pero se encuentra de manera constante en una comunidad o regin d) Los casos de una enfermedad aumentan en una determinada estacin del ao.

Enfermedad que por un tiempo afecta a un gran nmero de personas que viven en una misma regin. Enfermedad que por alguna temporada afecta a un pueblo o comarca, padecindola simultneamente un gran nmero de personas. Se denomina a la aparicin brusca de una enfermedad que se disemina rpidamente entre una poblacin determinada. A veces se la identifica errneamente como brote. La forma ms habitual es la producida por virus, debido a su hbito cambiante. Boffi H, lvarez-Herrera C. Contribucin al estudio de las enfermedades transmisibles mediante el uso de algunos mtodos estadsticos. Salud Argentina 1970;1:1397. [ 71.- El sndrome de Sheehan es? a) El sndrome de amenorrea galactorrea b) La hipotensin secundaria a la hemorragia post-parto c) La necrosis hipofisiaria postparto d) La apopleja de un tumor hipofisiario

La necrosis hipofisara posparto o Sndrome de Sheehan va precedida de hemorragia obsttrica que ocasiona colapso circulatorio intenso. Desde el punto de vista terico, la hipotensin grave predispone a la isquemia de la hipfisis aumentada de tamao. Por lo general se respeta la hipfisis posterior y la caracterstica clnica ms frecuente es la incapacidad para lactar por una produccin insuficiente de Prolactina (PR) Rev. Esp. MED. Quir. 2006; 11(2): 62-66. Sndrome de Sheehan. Rojas Ramn, Ramrez Manuel, Rodrigo Enrique.

72.- En un paciente con hipertensin arterial e hipokalemia debemos iniciar estudio de? e) Insuficiencia suprarrenal f) Hiperaldosteronismo primario g) Hipertiroidismo h) Hiperprolactinemia

Clnicamente es til la distincin entre hipokalemia con y sin hipertensin. Cuando la hipokalemia se asocia con hipertensin la medicin de renina y aldosterona plasmtica pueden ser tiles en el diagnostico diferencial, y los mecanismos por los cuales produciran hipokalemia serian: 1.- la aldosterona estimula la reabsorcin de sodio y de potasio en el tbulo colector 2. Los glucocorticoides en altas concentraciones se unen a los receptores de mineralocorticoides e imitan su accin fisiolgica.

Brenner and Rectors The Kidney, 6th Ed., Copyright 2000.W.B. Saunders Company. Approach to the diagnosis and treatment of hypokalemic and hyperkalemic states 73.- Una de las cardiopatas ms frecuentes en el sndrome de Down es? a). La transposicin de grandes vasos. b). La coartacin artica. c). Defecto atrioventricular. d). La comunicacin interauricular.

De los 275 nios estudiados, y que cursaron con cardiopata 160 (58%). Las cardiopatas que se presentaron con mayor frecuencia fueron la comunicacin interauricular (CIA), comunicacin interventricular (CIV) y persistencia del ductus arterioso (PDA) (90%); nicamente 14 casos (9%) correspondieron a defectos de la tabicacin auriculoventricular, a diferencia de lo observado en otros pases. La manifestacin clnica ms frecuente fue la insuficiencia cardaca. El 15% de los pacientes (n = 25) fallecieron, y las causas ms frecuentes fueron el choque sptico y cardiognico. De los pacientes con cardiopata congnita, el 4-10% se asocia a SD, y el 40-60% de los pacientes con este sndrome presenta cardiopata congnita. La malformacin cardiaca es la mayor causa de mortalidad en los primeros 2 aos de la vida , . En Mxico, las cardiopatas que se presentan con ms frecuencia en los nios con SD son la persistencia del ductus arterioso (PDA), la comunicacin interventricular (CIV) y la comunicacin interauricular (CIA), a diferencia de lo mencionado en pases anglosajones y europeos, donde los defectos de la tabicacin atrioventricular (D-AV) son los ms comunes (40-70%) y, entre estos, el canal atrioventricular parcial (CIA ostium primum con hendidura mitral) es el ms frecuente
5-10 56

56, Nmero 0 Maroto CM, Enrquez de Salamanca F, Herriz IS, Zabala JA.. Guas de prctica clnica de la Sociedad Espaola de Cardiologa en las cardiopatas congnitas ms frecuentes. Rev Esp Cardiol 2001;54:67-82.

74.-.- El frmaco antihipertensivo que da mayor cardioproteccin es? a) Diurticos b) Beta bloqueadotes c) Inhibidores de la ECA d) Vasodilatadores arteriales Los inhibidores de la ECA son medicamentos que ayudan a relajar los vasos sanguneos. Ello permite que la sangre circule ms fcilmente por los vasos sanguneos y reduce la presin sangunea. Al bajar la presin sangunea se reduce el riesgo de que se produzca un ataque cardaco o derrame cerebral, y se facilita el tratamiento de la insuficiencia cardiaca congestiva. La insuficiencia cardiaca congestiva se produce cuando el corazn est demasiado dbil o demasiado daado como para bombear la sangre adecuadamente. Los inhibidores de la ECA aparentemente reducen los daos producidos en el msculo cardaco despus de un ataque cardaco. Lisinopril, benazepril, enalapril, captopril, ramipril, quinapril y trandolapril son ejemplos de inhibidores de la eca (ACE). Cul es mejor para usted depende de su problema y estado de salud. HOPE Study Investigators. Effects of an angiotensin-converting enzyme inhibitor, Ramipril, on cardiovascular events in high-risk patients. New England Journal of Medicine 2000; 342: 145-153.

75.- La complicacin ms comn del infarto agudo al miocardio es? a) Arritmias ventriculares b) Insuficiencia cardiaca

c) Enema agudo pulmonar


d) Choque cardiognico COMPLICACIONES Las principales complicaciones que pueden aparecer en un paciente afectado de un infarto agudo de miocardio son las siguientes:

A) ARRITMIAS DE ORIGEN SINUSAL. 1. TAQUICARDIA SINUSAL:

ECG: Es un aumento de la frecuencia del ritmo sinusal superior a los 100 latidos por minuto y que casi nunca supera los 170 lat/min.

Fig.1 - Taquicardia sinusal.

Tratamiento: No suele precisar de tratamiento y lo fundamental es tratar la anomala de base. En caso de ser necesario se administraran preferentemente b-bloqueantes (propanolol, esmolol,...)[Los frmacos aqu presentados son aquellos que aparecen ms frecuentemente citados en la literatura, son una mera gua orientativa].

2. BRADICARDIA SINUSAL: ECG: Es la frecuencia del ritmo sinusal inferior a 60 lat/min. Cuando la frecuencia disminuye por debajo de 40 lat/min puede aparecer angina de pecho hipotensin, insuficiencia cardiaca o alteraciones de la conciencia.

Tratamiento: En pacientes asintomticos no requiere tratamiento. Si se asocia a compromiso hemodinmico o intomatologa se trata con atropina, isoprotenerol o implantacin de marcapasos ventricular.

3. FIBRILACIN AURICULAR: ECG: No se observa una actividad auricular clara (impulsos desordenados) y la respuesta ventricular es generalmente muy irregular (160-200 lat/min en ausencia de medicacin bloqueadora del nodo AV).

Fig.2 - Fibrilacin auricular con respuesta ventricular rpida.

Tratamiento: Depende de grado de respuesta ventricular y de las consecuencias hemodinmicas de la arritmia. - Si la respuesta ventricular es de ms de 140 lat/min y hay afectacin hemodinmica la cardioversin constituye el tratamiento de eleccin. - En pacientes hemodinmicamente estables se puede administrar digital, verapamilo, propanolol o esmolol para frenar la respuesta ventricular.

4. FLTER AURICULAR: ECG: El ritmo se caracteriza por una frecuencia auricular de entre 280-350 lat/min. La frecuencia ventricular varia segn el grado de bloqueo AV que suele ser 2:1 en pacientes no tratados. Las ondas caractersticas del flter se ven mejor en las derivaciones II, III, aVF y V1.

Fig.3 - Flutter auricular.

Tratamiento: En pacientes hemodinmicamente estables se puede administrar digital, verapamilo, propanolol o esmolol para frenar la respuesta ventricular. La indicacin de cardioversin es igual que para el caso de fibrilacin auricular.

B) ARRITMIAS DE ORIGEN VENTRICULAR. 1. RITMO DE LA UNIN AV O RITMO NODAL: ECG: Ritmo de fuga. Se produce en el fascculo de His cuando dejan de llegar impulsos procedentes del nodo sinusal. Frecuencia ventricular entre 40 - 60 lat/min siendo regular. Complejos QRS son iguales en el EKG que cuando existe ritmo sinusal. Tratamiento: Requiere observacin de la frecuencia cardiaca y no necesita tratamiento a no ser que esta sea excesivamente baja (<50 lat/min) o que no se mantenga una circulacin eficaz. En estos casos se debe considerar el uso de atropina o la implantacin de un marcapasos temporal.

2. TAQUICARDIA DE LA UNIN AV O NODAL: ECG: La taquicardia nodal conlleva una frecuencia ventricular de 100 - 140 lat/min. En este trastorno del ritmo, la automaticidad propia de la unin AV es superior a la del nodo sinusal. Puede ser debido a intoxicacin digitlica, fiebre reumtica o infarto de miocarido. Tratamiento: Administracin de potasio y retirada de la digital. Tambin puede ser eficaz el masaje del seno carotdeo. Si esto no resulta efectivo se puede emplear tratamiento antiarrtmico con lidocana. Cuando se est completamente seguro de que no se debe a intoxicacin por digital se puede emplear la cardioversin elctrica.

3. CONTRACCIONES VENTRICULARES PREMATURAS (CVP): ECG: Son despolarizaciones prematuras del ventrculo, por lo que se asocian a complejos QRS prematuros de una morfologa habitualmente distinta y con una duracin superior a 0,12 seg. La onda T generalmente es muy grande y de

direccin contraria a la deflexin mayor del complejo QRS. Este complejo no va precedido de onda T prematura, aunque se puede observar la onda P sinusal correspondiente a la actividad auricular. Tambin puede aparecer una onda P retrgrada detrs del complejo QRS, oculta o invisible en el EKG.

Fig.4 - Contracciones ventriculares prematuras.

Tratamiento: Las indicaciones para tratar a los pacientes ha sido objeto de controversia. En el caso de pacientes que sufren isquemia aguda los criterios generalmente aceptados son los siguientes: I.- Ms de 5 CVP/minuto. II.- Cayendo la Onda R de la CVP en la onda T de la pulsacin precedente (fenmeno de R sobre T). III.- Disparos de CVP. IV.- CVP multifocales.

La supresin aguda de las extrasstoles ventriculares se consigue de manera ptima con lidocana o procainamida IV. La supresin crnica se puede llevar a cabo con antiarrtmicos del grupo I. La amiodarona se reserva para aquellos casos que no respondan a los tratamientos convencionales.

4. RITMO IDIOVENTRICULAR ACELERADO (RIVA): ECG: Se caracteriza por tres o ms complejos QRS anchos y extravagantes que se presentan con una frecuencia similar a la frecuencia sinusal, siendo casi siempre inferior a 100 lat/min. Es particularmente frecuente despus del infarto de miocardio sobre todo despus de IAM inferior.

Tratamiento: Normalmente no requiere tratamiento. Si va acompaado de manifestaciones de hipoperfusin se debe administrar atropina para acelerar la frecuencia sinusal.

5. TAQUICARDIA VENTRICULAR (TV): ECG: Se define como la presencia de 3 o ms extrasstoles secuenciales a una frecuencia que oscila entre 120 - 150 lat/min. Es una arritmia peligrosa por su propensin a degenerar en fibrilacin ventricular. Cuando la TV dura ms de 30 seg o causa un colapso hemodinmico que requiere su terminacin se denomina TV sostenida. La TV no sostenida dura menos de 30 seg y se detiene espontneamente.

Fig.5 - Taquicardia ventricular.

Tratamiento: Si la TV mantenida se asocia con hipotensin, insuficiencia cardiaca o angina de pecho est indicada la cardioversin inmediata (comenzando por 200 J y aumentando hasta 360 J si no es eficaz). si la TV mantenida es bien tolerada por el paciente, la administracin de los siguientes antiarrtmicos puede poner fin a la arritmia: lidocana (bolo IV [1 mg/kg] seguido de una perfusin IV); procainamida [dosis de saturacin de 1500 mg en 150 ml de SF o SG5% seguido de perfusin IV); bretilio [dosis de 5-10 mg/kg en 8-10 minutos seguido de infusin IV continua]. Conviene mantener el tratamiento con estos frmacos durante algunos das.

6. DISOCIACIN AURICULO-VENTRICULAR: ECG: Las aurculas y los ventrculos son controlados por dos marcapasos separados e independientes. Puede asociarse con taquicardias supraventriculares, distintos grados de bloqueo AV y taquicardia ventricular.

Tratamiento: Si el paciente tolera el ritmo puede no ser necesario su tratamiento. La correccin de la enfermedad o del trastorno del ritmo subyacente subsanar con frecuencia la disociacin AV.

7. FIBRILACIN VENTRICULAR (FV): ECG: Trastorno del ritmo ventricular en el que los impulsos ocurren de un modo irregular y a una frecuencia, en general, elevada, por lo que la contraccin cardiaca es incoordinada y el volumen de expulsin nulo. Se observa en el ECG un ritmo completamente irregular en anchura amplitud y frecuencia. Tratamiento: Se debe emplear la desfibrilacin elctrica como tratamiento de primera eleccin lo ms tempranamente posible desde el inicio de la FV. Un retraso hace ms dificultosa la reversin del ritmo a sinusal.

C) TRASTORNOS DE CONDUCCIN. BLOQUEO AURICULO-VENTRICLUAR: I.-Bloqueo A-V de 1 grado: Se caracteriza por una prolongacin del intervalo PR. II.-Bloqueo A-V de 2 grado: Se caracteriza por el fallo intermitente de la llegada del impulso al ventrculo, con lo que se produce una omisin de latidos ventriculares. Existen dos tipos: bloqueo de Mobitz tipo I o bloqueo de Wenckebach (en este caso el intervalo PR aumenta progresivamente hasta que se bloquea un impulso auricular y se produce la falta de un latido ventricular; se produce el defecto de conduccin por encima del fascculo de His; la cantidad de contracciones auriculares entre los fallos de contraccin ventricular pueden variar considerablemente) y bloqueo de Mobitz tipo II (se caracteriza por un intervalo PR fijo producindose omisin de latidos, los latidos pueden faltar irregularmente o siguiendo un modelo regular [2:1, 3:1 o 4:1 son los ms frecuentes]; el origen de este bloqueo es un trastorno de conduccin en el seno o por debajo del fascculo de His) III.-Bloqueo A-V de 3 grado: Se caracteriza por despolarizaciones auriculares que nunca son transmitidas al ventrculo. Los ventrculos son despolarizados por un marcapasos independiente, en general nodal o infranodal. La frecuencia

ventricular es generalmente de 30 - 50 lat/min.

Fig.5 - Bloqueo A-V de 3 grado.

BLOQUEO DE RAMA FASCICULAR: ECG: Los bloqueos de rama fascicular reflejan trastornos de conduccin en regiones especficas de las ramas fasciculares derecha o izquierda, la divisin posterior o anterior del fascculo izquierdo o ambos. El diagnstico se realiza por criterios electrocardiogrficos de acuerdo con la prolongacin del complejo QRS y el desplazamiento del eje. Tratamiento: Los pacientes con bloqueo crnico de rama fascicular o bifascicular son tratados si se documenta un bloqueo A-V de 3 grado o se sospecha fuertemente. El tratamiento consiste en la implantacin de un marcapasos permanente.

SNDROMES DE PREEXCITACIN: ECG: Se caracterizan por la despolarizacin prematura de una parte del msculo ventricular. Se reconocen por el EKG. El sndrome de Wolff-Parkinson-White se caracteriza por la existencia de ondas P normales, intervalo PR de 0,11 seg. o menos, una mancha inicial de QRS (onda Delta) y una prolongacin del QRS. El sndrome de Lown-Ganong-Levine se caracteriza slo por un intervalo PR acortado. Se ha comprobado la presencia de una conduccin anmala que elude el nodo AV. Los pacientes suelen ser asintomticos o presentar palpitaciones o episodios sincopales.

Fig.6 - Sndrome de Wolff-Parkinson-White.

2. INSUFICIENCIA CARDIACA: El factor determinante de la insuficiencia cardiaca es la extensin del infarto. El gasto cardiaco y el volumen sistlico quedan comprometidos cuando se afecta el 20% del miocardio. La aparicin de shock cardiognico se produce con una prdida acumulativa de, al menos, el 40% del miocardio ventricular izquierdo y es la principal causa de muerte en la fase aguda del infarto de miocardio. 3. COMPLICACIONES MECNICAS: Aparecen como consecuencia de la prdida de integridad de la estructura del corazn: ruptura de pared externa, ruptura del tabique interventricular y ruptura del msculo papilar. 4. EXTENSIN DE LA ISQUEMIA: Angor postinfarto: se considera angina postinfarto aquella que aparece dentro del primer mes posterior al infarto. Si no se controla farmacolgicamente est indicada la realizacin de una coronariografa, si se controla adecuadamente mediante frmacos la realizacin de esta prueba puede demorarse.

Banzo I, Montero A, Uriarte I, Vallina N, Hernandez A, Guede C, et al. Coronary artery occlusion and myocardial infarction: a seldom encountered complication of blunt chest trauma. Clin Nucl Med 1999 24: 94-6.

76.- La triada de Virchow en la etiologa de trombo embolia pulmonar esta constituida por: a) Estasis Venosa, Hipercoagulabilidad y lesin del endotelio. b) Estasis Venosa, lesin del endotelio e hipoxemia. c) Hipoxemia, hipercoagulabilidad y tos d) Tos Seca

Los 3 componentes de la triada de Virchow son estasis venosa, hipercoagulabilidad y lesin del endotelio vascular que predisponen al desarrollo de la trombosis venosa profunda, la que a su vez condiciona trombo embolia pulmonar. Fishman, A; Manual de Enfermedades pulmonares, tercera edicin, Mc Graw Hill, pags 40-41, 2004.

77.- Patologas que incluyen el trmino EPOC: a) b) c) d) Bronquitis Aguda Bronquitis Crnica y Enfisema. Asma Bronquial Bronquiolitis

El EPOC se define como un estado de enfermedad caracterizado por la presencia de obstruccin del flujo areo, parcialmente reversible. Esta definicin excluye al asma bronquial. Fishman, A; Manual de Enfermedades pulmonares, tercera edicin, Mc Graw Hill, Pgs. 129-130, 2004.

78.- La imagen histolgica de coilocitos sugiere infeccin por? a) b) c) d) Herpes virus tipo 2 Citomegalovirus Virus del papiloma humano Vaginosis bacteriana.

Histologa de la NIC 1: Obsrvese que las clulas displsicas estn confinadas en el tercio inferior del epitelio. Los coilocitos indicados por flechas se observan sobre todo en los estratos epiteliales superiores

Schlecht, N.F., Kulaga, S., Robitaille, J., Ferreira, S., Santos, M., Miyamura, R.A., Duarte-Franco, E., Rohan, T.E., Ferenczy, A., Villa, L.L., & Franco, E.L. (2002) Persistent Human Papillomavirus Infection as a Predictor of Cervical Intraepithelial Neoplasia. JAMA, 286, 3106-3114

79.- Posible diagnstico en un paciente masculino de 17 aos de edad que desconoce sus antecedentes familiares. Desarrolla dolor abdominal progresivo, posterior a un traumatismo directo en la regin lumbar; se documenta un hematoma en el msculo posas izquierdo La Bh con 11 gr. de HB, reticulocitos 35, LEUCOCITOS DE 13500, 350 mil plaquetas, TTP: 60/35, TP: 12/12, TT: 26/26 fibringeno 350 MG/dl. Cul es su sospecha diagnstica? a) Enfermedad de Von Willebrand b) Hemofilia c) Trombocitopata d) Deficiencia de factor XII

Cules son los sntomas de la hemofilia?

El sntoma ms comn de la hemofilia es la hemorragia incontrolable y excesiva por causa del factor de coagulacin que falta o est en bajos niveles en la sangre. Puede producirse una hemorragia incluso cuando no haya ninguna lesin. La mayora de las veces se produce en las articulaciones y en la cabeza. A continuacin, se enumeran los sntomas ms frecuentes de la hemofilia. Sin embargo, cada individuo puede experimentarlos de una forma diferente. Los sntomas pueden incluir: Equimosis (Moretones) Pueden producirse moretones por pequeos accidentes, que pueden producir un hematoma grande (una acumulacin de sangre debajo de la piel que causa inflamacin). Sangra con facilidad La tendencia a sangrar por la nariz, la boca y las encas por un traumatismo sin importancia, cepillarse los dientes y, o trabajo dental es a menudo una indicacin de hemofilia. Hemorragia en una articulacin La hemartrosis (hemorragia en una articulacin) puede producir dolor, inmovilidad y, con el tiempo, deformacin si no se trata mdicamente de la forma adecuada. sta es la zona ms comn de complicaciones debido a la hemorragia por hemofilia. Estas hemorragias de las articulaciones pueden producir la artritis crnica y dolorosa, deformaciones y parlisis si se repiten. Hemorragia en los msculos La hemorragia en los msculos puede causar hinchazn, dolor y enrojecimiento. La hinchazn por el exceso de sangre en estas zonas puede producir un aumento de la presin en los tejidos y nervios de la zona, provocando dao y, o deformacin permanente.

Hemorragia por lesiones o hemorragia cerebral La hemorragia por lesin o espontnea en el cerebro es la causa ms comn de muerte en los nios que tienen hemofilia y la complicacin hemorrgica ms grave. Otras fuentes de hemorragia La sangre en la orina o en las heces puede ser tambin un sntoma de hemofilia.

Los sntomas de la hemofilia pueden parecerse a los de otros trastornos de la sangre o problemas mdicos. Siempre consulte a su mdico para el diagnstico.

Roberts HR. Hemophilia A and Hemophilia B. In: Lichtman, MA, et al., eds. Williams Hematology. 7th ed. The McGraw-Hill Companies, Inc.; 2006:chap 115.

80.- La equimosis periorbitaria en paciente con trauma facial, traduce fractura de: a) b) c) d) Piso posterior Piso medio Piso anterior Macizo facial

En la valoracin ocular se deben tener en cuenta las heridas de los tejidos blandos de prpado, crnea y conjuntiva. El signo de mapache (equimosis periorbitaria bilateral) se encuentra frecuentemente asociado a las fracturas de la base anterior del crneo. (Figura 3).

Figura 3. Equimosis periorbitaria luego de trauma facial contundente.

En las heridas del prpado la localizacin es de vital importancia y es necesario anotar cuidadosamente el sitio, si se compromete el borde libre o si se lesiona el lugar de paso de la va lagrimal. El tono ocular ayudar a definir si hubo herida abierta del ojo. La presencia de enoftalmo (hundimiento del globo ocular) alertar sobre la posibilidad de una herida abierta del ojo o una fractura de las paredes orbitarias. El hipoftalmos (descenso del ojo en el eje vertical) puede estar relacionado con fractura del piso de la

rbita o con fractura en el sitio donde se insertan los ligamentos suspensorios del globo ocular (tubrculo de Whitnall, ligamento de Lookwood). H. Ric Harnsberger. 2004. ISBN 848174753x Translation of PocketRadiologist - Head & Neck: Top 100 Diagnoses.

81.- El trastorno por stress postraumtico se caracteriza por: a) Ser un cuadro agudo postraumtico que dura menos de un mes. b) Nunca se presenta con pesadillas. c) Se presenta en el 25 % de la poblacin en algn momento de la vida. d) El tratamiento farmacolgico esta contraindicado. Pautas para el diagnstico Este trastorno no debe ser diagnosticado a menos que no est totalmente claro que ha aparecido dentro de los seis meses posteriores a un hecho traumtico de excepcional intensidad. Un diagnostico "probable" podra an ser posible si el lapso entre el hecho y el comienzo de los sntomas es mayor de seis meses, con tal de que las manifestaciones clnicas sean tpicas y no sea verosmil ningn otro diagnstico alternativo (por ejemplo, trastorno de ansiedad, trastorno obsesivo-compulsivo o episodio depresivo). Adems del trauma, deben estar presentes evocaciones o representaciones del acontecimiento en forma de recuerdos o imgenes durante la vigilia o de ensueos reiterados pero nunca se presentaran acompaados de pesadillas. Tambin suelen estar presentes, pero no son esenciales para el diagnstico, desapego emocional claro, con embotamiento afectivo y la evitacin de estmulos que podran reavivar el recuerdo del trauma. Los sntomas vegetativos, los trastornos del estado de nimo y el comportamiento anormal contribuyen tambin al diagnstico, pero no son de importancia capital para el mismo. Friedman MJ (1990) Interrelationships between biological mechanism and pharmacology of post-traumatic stress disorder, In: Wolfe ME, Mosnaim AD, eds, Postraumatic Stress Disorder: Etiology, Phenomenology and Treatment, Washington DC: American Psychiatric Press; 1990:204-225

82.- En relacin al trastorno por dficit de atencin e hiperactividad, indica la opcin correcta: a) Se presenta en el 20% de la poblacin.

b) Ms comn en mujeres. c) El diagnostico es por presentar mas de 2 sntomas en dos reas de afectacin (atencin, hiperactividad e impulsividad). d) El tratamiento farmacolgico incluye psicoestimulantes y antidepresivos. Los frmacos de primera lnea son los psicoestimulantes y aunque est comprobado fehacientemente los excelentes beneficios de la doss dependente a corto trmino que se obtienen en los nios y en algunos adolescentes con SH/DA, estudios longitudinales no han podido demostrar que sus efectos sean permanentes4. Los ms usados son la anfetamina, el metilfenidato y la pemolina. la primera puede adquirirse en nuestro medio con muchas restricciones (y prejuicios), el segundo ya ha retornado al mercado peruano y la ltima, de hecho, nunca ha llegado a ste. Su efectividad cubre el 80% de los pacientes tratados y ella es prcticamente imposible de predecir, al igual que sus frecuentes efectos secundarios, en cada individuo. Wilens TE, Spencer TJ, Biederman J. Pharmacotherapy of ADHI) in adults. CNS Drugs 1998; 9: 347-56. En: World Federation of Neurology: World Neurol 1998; 13(3): 12.

83.-. El signo clnico para diagnostico de escoliosis se conoce como: a) Signo de Cajn b) Signo de Allis c) Signo de McMurray d) Signo de Adams Esta rotacin se detecta clnicamente con el test de Adams, cuando el paciente flexiona su columna, existe asimetra de la posicin de la parrilla costal y/o los flancos lumbares Skinner, H. Diagnstico y tratamiento en Ortopedia. Ed. Manual Moderno. Mxico, 2004. pp. 256

Caso Clnico (relaciona preguntas 84, 85, 86, 87, 88,89) Paciente masculino con antecedentes de cardiopata isqumica hipertensiva, el cual se presenta a sala de urgencias por dolor intenso de la extremidad plvica derecha de 2 horas de evolucin. a la exploracin fsica: lo encontramos angustiado con fascies de dolor, TA 150/90, FC 120X FR 18X Temp. 36oc. soplo carotdeo bilateral, ruidos cardiacos arrtmicos y de buena intensidad y sin fenmenos agregados, ambos hemitrax bien ventilados, abdomen blando, depresible y no doloroso, peristalsis presente, se ausculta soplo sistlico en mesogastrio, extremidades torcicas con pulsos normales hasta distales, extremidad plvica izquierda con pulso femoral con soplo sistlico, pulso poplteo, tibial posterior y anterior disminuidos, llenado capilar inmediato, extremidad plvica derecha sin pulso femoral ni popliteo ni tibiales, plida,

hipotrmica, con sensibilidad disminuida, con movimientos normales de los dedos y la articulacin del tobillo.

84.- En el caso clnico anterior usted qu diagnstico integrara? a) b) c) d) Insuficiencia arterial crnica compensada Insuficiencia arterial crnica agudizada Enfermedad de Leriche Isquemia critica del miembro plvico

El sndrome de isquemia crnica es el conjunto de sntomas y signos producidos por una inadecuada irrigacin arterial que, de forma progresiva, se ha establecido en las extremidades. Afecta ms frecuentemente a varones, dndose la mayor incidencia en el sexto y sptimo decenio de la vida. Se consideran factores de riesgo la hipertensin, la hipercolesterolemia, el tabaco y la diabetes mellitus entre otros.

Se entiende por sndrome de Leriche a la patologa obstructiva de la bifurcacin aortoilaca. los sntomas tpicos consisten en claudicacin intermitente que afecta la pantorrilla, muslos y glteos. La isquemia crtica equivaldra los grados III y IV de la clasificacin de Fontaine (III- Dolor en reposo y IV- Lesiones trficas). Manual CTO 5 edicin, Cardiologa, pg 84- 86 85.- La principal causa etiolgica de esta enfermedad es:

a) b) c) d)

Lesin endotelial y formacin de placa de ateroma Combinacin de factores de riesgo para ateroesclerosis Desarrollo de reas de acinesia cardiaca secundarias a isquemia Doble lesin valvular pulmonar

El sndrome de isquemia crnica es el conjunto de sntomas y signos producidos por una inadecuada irrigacin arterial que, de forma progresiva, se ha establecido en las extremidades. ETIOLOGA: Sin duda alguna, la causa ms frecuente de isquemia crnica de los miembros es la aterosclerosis. Esta enfermedad constituye hoy en da una de las plagas modernas, siendo la morbilidad y mortalidad de la misma impresionante. Afecta ms frecuentemente a varones, dndose la mayor incidencia en el sexto y sptimo decenio de la vida. Se consideran factores de riesgo la hipertensin, la hipercolesterolemia,el tabaco y la diabetes mellitus entre otros.

La embolia arterial aguda se sospecha cuando existe una fuente cardaca potencial, por lo que es prctica habitual la realizacin de un ECG en todos los pacientes con oclusin arterial aguda. La isquemia de inicio agudo tambin sugiere un origen emblico. La ausencia de pulsos en una extremidad y su conservacin en la contralateral apunta a una etiologa emblica. Braunwald E. Kasper D, et al. Harrison: Principios de Medicina Interna. Editorial Mc GraHill 16a edicin. Mxico, 2006. pp 1645. Manual CTO 5 edicin, Cardiologa, pg 84- 88

86.- De acuerdo a las caractersticas clnicas del paciente podemos concluir que el paciente: a) b) c) d) Se encuentra en una fase II de Fontaine Tiene datos de isquemia parcial compensada Hay datos de isquemia irreversible Presenta isquemia clase II de Rutherford

Manual CTO 5 edicin, Cardiologa, pg 84- 86

87.- Con base en el diagnstico anterior el tratamiento inmediato a seguir es: a) b) c) d) Realizacin de arteriografa selectiva de aorta Digitalizacin del paciente y valoracin por cardiologa Colocacin de bloqueo peridural teraputico Heparinizacin y embolectoma femoral

Una vez establecido el diagnstico, el paciente debe ser anticoagulado con heparina por va intravenosa para evitar la propagacin del trombo. En casos de isquemia grave de instauracin reciente, y sobre todo cuando existe riesgo para la viabilidad del miembro, esta indicada la intervencin inmediata para asegurar la reperfusin. Para restablecer con prontitud el flujo sanguneo se utiliza tromboembolectoma endovascular o quirrgica o procedimientos de derivacin, particularmente cuando est obstruido un vaso proximal de gran calibre.

La localizacin de la obstruccin es indispensable de cara al tratamiento. Este diagnstico topogrfico se hace por la localizacin del dolor y por la presencia o ausencia de pulsos, ya que faltarn por debajo de la lesin. El doppler suele bastar para facilitar un diagnstico del nivel lesional, pero hay ocasiones en las que se debe recurrir a la arteriografa. La arteriografa consiste en el registro de la imagen radiolgica de las arterias tras la administracin de un contraste. Por ser una exploracin invasiva no debe emplearse como prueba de rutina, pero es un mtodo diagnstico muy valioso para que el cirujano planifique la

estrategia quirrgica, por lo que se lleva a cabo antes de una posible intervencin de revascularizacin.

Braunwald E. Kasper D, et al. Harrison: Principios de Medicina Interna. Editorial Mc Graw Hill, 16a edicin. Mxico, 2006. pp 1645. Manual CTO 5 edicin, Cardiologa, pg 84

88.-. La complicacin ms frecuente posterior al tratamiento es: a) b) c) d) Infeccin del sitio de puncin Tromboembolismo pulmonar paradjico Isquemia irreversible de la extremidad Sndrome de repercusin

La revascularizacin con xito de una extremidad que ha sufrido por un perodo de tiempo prolongado la ausencia o disminucin del flujo arterial, produce lesin de las clulas musculares an viables. Esta lesin por isquemia- reperfusin con mioglobinuria, hiperkalemia y acidosis metablica fue descripta en 1960 por Haimovici, en lesiones por aplastamiento. Originariamente descrito como sndrome de reperfusin o bien ltimamente como sndrome mionefroptico metablico que se da al restablecer el flujo sanguneo con la consiguiente restauracin del aporte energtico y la liberacin de metabolitos txicos hacia la circulacin, trayendo esto ltimo serias consecuencias metablicas por la reintroduccin de oxgeno con la produccin de radicales libres, neutrfilos, factor activador plaquetario y otros agentes proinflamatorios, adems del aumento enzimtico por lisis muscular de la CPK, LDH y GOT. Estos mismos inducen lesin en los tejidos locales llevando a una mortalidad elevada sobre todo por el cortejo sintomtico que produce insuficiencia renal aguda por necrosis tubular aguda, arritmias cardacas por hiperkalemia y acidosis metablica por la anoxia tisular favorecida por la obstruccin arterial. Callum Ken- Bradbury Andrew- ABC of arterial and venoud disease- Clinical review Acute limb ischaemia BMJ 2000; 320: 764-767 (March 18

89.- El manejo de esta complicacin debe consistir en: a) Antibiticos de amplio espectro y desbridacin de la herida quirrgica b) Colocacin de filtro de greenfield en vena cava inferior c) Amputacin supracondlea de la extremidad d) Alcalinizacin de la orina, manitol y alopurinol.

Originariamente descrito como sndrome de reperfusin, que se da al restablecer el flujo sanguneo con la consiguiente restauracin del aporte energtico y la liberacin de metabolitos txicos hacia la circulacin, trayendo esto ltimo serias consecuencias metablicas por la reintroduccin de oxgeno con la produccin de radicales libres, neutrfilos, factor activador plaquetario y otros agentes proinflamatorios, adems del aumento enzimtico por lisis muscular de la CPK, LDH y GOT. Estos mismos inducen lesin en los tejidos locales llevando a una mortalidad elevada sobre todo por el cortejo sintomtico que produce insuficiencia renal aguda por necrosis tubular aguda, arritmias cardacas por hiperkalemia y acidosis metablica por la anoxia tisular favorecida por la obstruccin arterial. Este sndrome debe tratarse con frmacos barredores de radicales libres, alcalinizacin de la orina, inclusive antes de iniciar la revascularizacin, con lo cual se disminuye significativamente la morbilidad. Callum Ken- Bradbury Andrew- ABC of arterial and venoud disease- Clinical review Acute limb ischaemia BMJ 2000; 320: 764-767 (March 18)

90.-. El desarrollo de sndrome compartamental en este paciente est en relacin a : a) b) c) d) El tiempo de isquemia de la extremidad El desarrollo de sndrome de reperfusin El desarrollo de trombosis venosa profunda El aumento en la presin de perfusin postquirrgico

Se define como el cuadro clnico que se establece cuando la presin tisular en el interior de un compartimento osteofascial aumenta hasta ocluir la circulacin capilar. El aumento de presin puede deberse a circunstancias que aumenten el contenido del compartimento (edema resultante de un traumatismo, quemadura o reperfusin postisquemia; hematoma procedente de un foco de fractura. La isquemia consecutiva al aumento de presin desencadena fenmenos de necrosis muscular y nerviosa. Inicialmente el paciente refiere un dolor desproporcionado para la lesin que presenta; el dolor aumenta con el estiramiento pasivo de los msculos contenidos en dicho compartimento Posteriormente aparecen alteraciones sensitivas. El pulso distal puede y suele estar conservado Mendoza CA, Manzo CHA Sndrome compartimental en extremidades.Conceptos actuales Cir Gen 2003; 25 (4): 342-348

91.- Posterior a tratamiento quirrgico el paciente refiere dolor de mayor intensidad que antes de su ciruga y adormecimiento de los dedos, al revisarlo se encuentra con edema severo de la extremidad con pulsos apenas perceptibles y llenado capilar retardado as usted decide: a) b) c) d) realizar exploracin arterial quirrgica de urgencia aumentar las dosis de heparina y elevar la extremidad suspender la heparina y drenaje de hematoma realizar fasciotomias en la cama del paciente.

Si el cuadro no cede, es necesaria la apertura quirrgica urgente del compartimento o compartimentos afectos mediante fasciotoma Manual CTO 5 edicin, Traumatologa y Ortopedia, pg 6-7. 92.- La diferencia principal entre los estudios observacionales y los experimentales es que en los estudios experimentales? a) Los grupos de estudio y de control son del mismo tamao b) El investigador determina cuales sujetos recibirn la exposicin c) Los grupos de estudio y de control son compatibles y tienen las mismas caractersticas d) Se usan controles. Aunque existen mltiples clasificaciones de los diseos de investigacin, la mayora de los autores estn de acuerdo en la existencia de dos grandes tipos: experimentales y observacionales (no experimentales). La diferencia bsica estriba en el mtodo empleado para asignar a los sujetos del estudio a la exposicin o intervencin.

Ruiz M. A. Epidemiologa Clnica, 1. ED. 2004; Pg.: 234

93.- Para valorar que tan fuerte es la asociacin entre una exposicin y una enfermedad, cul sera la medida de asociacin a calcular? a) b) c) d) Riesgo relativo Tasa de mortalidad Incidencia Prevalencia

El Riesgo Relativo mide la fortaleza de una asociacin entre un factor y un cierto resultado final; de este modo, un Riesgo Relativo orienta hacia causalidad y es til para investigar el origen de una enfermedad,

Morton R. F. Bioestadstica y Epidemiologa, Interamericana, 3. Ed. 1993; pg: 36

94.- De las siguientes opciones, cual es correcta para el tratamiento correcto inicial del paciente con infeccin por VIH: a) Inmunoestimulantes mas un antiviral b) Dos inhibidores de proteasas, ms un inhibidor de transcriptasa reversa anlogo de nuclesidos. c) Dos inhibidores de transcriptasa reversa no anlogos de nuclosidos, y un inhibidor de proteasa. d) Dos inhibidores de transcriptasa reversa anlogos de nuclesidos y un inhibidor de proteasa.

El tratamiento inicial para el paciente con VIH debe seguir dos caminos: o utilizar inhibidores de proteasa o utilizar inhibidores de transcriptasa reverse no anlogos de nuclesidos. De estos grupos se elige un compuesto, el cual se debe acompaar con una base de dos inhibidores de transcriptasa reversa anlogos de nuclosidos. Kasper DL, Braunwald E, Fauci AS, Hauser SL, Longo DL, Jameson JL. Harrisons Principles of Internal Medicine. McGraw Hill. 16 Ed. 1076 p.

95.- Dada su cobertura cul de los siguientes antibiticos elegira para el tratamiento emprico de un absceso pulmonar en un paciente que probablemente haya presentado broncoaspiracin? a) b) c) d) Clindamicina Isoniazida Amikacina Claritromicina

La mayor parte de los abscesos en esta condicin estn ocasionados por cocos gram positivos anaerobios, para los cuales la Clindamicina tiene muy buena cobertura, adems de que sus propiedades farmacolgicas le confieren importante penetracin en la zona del absceso. Kasper DL, Braunwald E, Fauci AS, Hauser SL, Longo DL, Jameson JL. Harrisons Principles of Internal Medicine. McGraw Hill. 16 Ed. 1536 p.

96.- Paciente con infeccin por VIH, diagnosticado hace 2 aos, asintomtico. Acude a consultarlo porque no ha podido ir con el infectlogo que lo ha revisado en los ltimos 6

meses y quien haba solicitado como control una carga viral que se report con 100,000 copias, y una cuenta de CD4 de 33 clulas. Usted le recomienda. a) b) c) d) Inicio de tratamiento con AZT y 3TC. Ir con su infectlogo. Repetir el estudio. Iniciar TMP/SMZ y claritromicina.

El conteo de CD4 tan bajo pone al paciente en riesgo de desarrollar un evento por infeccin oportunista potencialmente mortal en el siguiente ao por arriba del 80%. Es indispensable, en tanto recibe atencin por el especialista, iniciar tratamiento profilctico contra Pneumocystis jiroveci y Mycobacterium avium-intracellulare. Kasper DL, Braunwald E, Fauci AS, Hauser SL, Longo DL, Jameson JL. Harrisons Principles of Internal Medicine. McGraw Hill. 16 Ed. 1076 p.

97.- Cul de los siguientes antivirales se utiliza en el tratamiento de hepatitis C? a) b) c) d) Adefovir Didanosina Oseltamivir Ribavirina

La combinacin de interfern y Ribavirina ha sido el tratamiento que ha demostrado tener la mayor proporcin de pacientes con respuesta viral sostenida en esta infeccin. Kasper DL, Braunwald E, Fauci AS, Hauser SL, Longo DL, Jameson JL. Harrisons Principles of Internal Medicine. McGraw Hill. 16 Ed. 1835 p.

98.- Es un hallazgo de la uropata obstructiva baja en la primera etapa: a) Vejiga poco contrctil b) Vejiga plida y poco vascularizada c) Contracciones por arriba de 20 cm de agua d) Esfnter hipotnico La inyeccin de un diurtico constituye una ayuda para definir mejor situaciones imprecisas. El choque de agua o la administracin de un diurtico(furosemida) provocan una respuesta tanto ms evidente cuanto mejor funcin renal tenga el paciente. Aclaramientos por debajo de10-20 cc/min difcilmente van a provocar esa respuesta y, por tanto, no nos permiten excluir la obstruccin. La prueba se convierte, por tanto, en vlida para el diagnstico y pronstico si la funcin renal es buena. El aumento de todos los dimetros de las cavidades (Figs. 12 y 13), tras la inyeccin diurtica, puede tener el reflejo clnico del desencadenamiento de dolor renal en aquellos casos obstruidos.

OBSTRUCCIN URINARIA J.J. BALLESTEROS SAMPOL Servicio de Urologa. Hospital Ntra. Sra. de la Esperanza. Barcelona. PALABRAS CLAVE: Uropata obstructiva. Fisiopatologa. Tratamiento. Tcnicas diagnsticas. KEY WORDS: Obstructive uropathy. Pathophysiology. Management. Diagnostic techniques.

99.- Complicacin que puede surgir por la enfermedad o por el tratamiento en casos de tuberculosis urinaria: e) Gangrena gaseosa f) Divertculos ureterales g) Cncer de urotelio h) Estenosis ureteral Durante la fase de diseminacin hematgena (primoinfeccin) se produce siembra de bacilos en ambos riones en el 90% de los casos, sin embargo, la enfermedad clnica generalmente es unilateral. El perodo de latencia entre la siembra y la enfermedad clnica oscila entre 10 y 40 aos, afectando principalmente a pacientes por debajo de los 50 aos. La lesin inicial microscpica se localiza en los glomrulos en forma de granulomas microscpicos. Al avanzar la enfermedad se produce afectacin ms distal hasta la aparicin de una papilitis necrotizante, momento en el cual puede ya existir paso de bacilos a la va excretora, donde por procesos inflamatorios ocasionar estenosis a nivel de los infundbulos caliciales, pelvis y urter, con hidronefrosis secundaria. Manual CTO 7 edicin, Urologa, pg5 100.- En la insercin anormal placentaria, cuando se infiltra hasta la serosa uterina se le denomina: a) Percreta b) Increta c) Acreta d) Percreta dextruens

En un embarazo normal, la placenta se adhiere a la pared uterina lejos del cuello del tero. Placenta accreta se refiere a una placenta que se fija a demasiada profundidad y con demasiada firmeza en la pared del tero. Placenta increta es una placenta que se fija con una profundidad an mayor en la pared uterina. Placenta percreta es una placenta que se fija a travs del tero y a veces se extiende hasta los rganos circundantes, como la vejiga. Estos trastornos se producen en aproximadamente 1 de cada 2.500 embarazos. Suelen

causar hemorragia vaginal en el tercer trimestre y con frecuencia resultan en un parto prematuro. Dado que la placenta no puede separarse fcilmente de la pared del tero despus del parto, por lo general se extirpa quirrgicamente. Suele ser necesario realizar una histerectoma (la extirpacin del tero), aunque pueden utilizarse otros procedimientos quirrgicos para salvar el tero. Hay poco que una mujer con una de estas condiciones pueda hacer para tratarla o prevenirla. Si se le diagnostica placenta accreta antes del parto (mediante un ultrasonido), puede planificarse el parto y considerarse un procedimiento quirrgico para salvar el tero. Esto es algo que debe discutir con su mdico, especialmente si desea tener otro hijo en el futuro. Pero si la condicin es grave, puede que no sea posible.

Perucca E, Domnguez C, Yahng Ch, Garca R. Placenta previa percreta con invasin vesical. Rev Chil Obstet Ginecol 1997; 62(3): 206-10.

Abbas F, Talati J, Wasti S et al. Placenta percreta with bladder invasion as a cause of life threatening hemorrhage. J Urol 2000; 164: 1270-4.

Anda mungkin juga menyukai